SlideShare a Scribd company logo
1 of 42
Slide 1 of 42
Type - 1
1. If in a certain code, LUTE is written as MUTE and
FATE is written as GATE, then how will BLUE be
written in that code ?
(a) CLUE (b) GLUE
(c) FLUE (d) SLUE
Sol. (a).
The first letter of the word is moved one step forward
to obtain the first letter of the code, while the other
letters remain unaltered.
2. If FISH is written as EHRG in a certain code, how
would JUNGLE be written in that code ?
(a) ITMFKD (b) ITNFKD
(c) KVOHMF (d) TIMFKD
Sol. (a).
Each letter in the word is moved one step backward
to obtain the corresponding letter of the code.
3. In a certain code, ROAD is written as URDG. How
is SWAN written in that code ?
(a) VXDQ (b) VZDQ
(c) VZCP (d) UXDQ
Sol. (b).
Each letter in the word is moved three steps forward
to obtain the corresponding letter of the code.
4. In a certain code, FAVOUR is written as EBUPTS.
How is DANGER written in that code ?
(a) CBFFDS (b) CBMHDS
(c) EBFHDS (d) EBHHFS
Sol. (b).
The first, third and fifth letters are each moved one
step backward, while the second, fourth and sixth
letters are each moved one step forward to obtain the
corresponding letters of the code.
Slide 2 of 42
5. In a certain code, PRODUCTIONS id written as
QQPCVEUHPMT. How is ORIENTATION
written in that code ?
(a) PQJDOVBSJNO (b) PQJDOUBUJPO
(c) PSJFOVBSJNO (d) NSHFMVBSJNO
(e) None of these
Sol. (a).
The first, third, fifth, seventh, ninth and eleventh
letters in the word are each moved one step forward ;
the second, fourth, eight and tenth letters are each
moved one step backward, while the middle (i.e.
sixth) letter is moved two steps forward to obtain the
corresponding letters of the code.
6. In a certain code, BASIC is written as DDULE.
How is LEADER written in that code ?
(a) NGCFGT (b) NHCGGU
(c) OGDFHT (d) OHDGHU
Sol. (b).
The letters at the odd-numbered positions in the
word are each moved two steps forward while those
at the even-numbered positions are each moved three
steps forward to obtain the corresponding letters of
the code.
7. If in a certain language, MIRACLE is coded as
NKUEHRL, then how is GAMVLE coded in that
language ?
(a) JDOCMF (b) CLEMNK
(c) HCPFQK (d) AELGMN
Sol. (c).
The first, second, third, fourth, fifth, sixth and
seventh letters in the word are moved one, two, three,
four, five, six and seven steps forward respectively to
obtain the corresponding letters of the code.
Slide 3 of 42
9. In a certain code, SPRING is written as UNUFRC.
How will the word MOBILE be written in that code
language ?
(a) KQEFPA (b) OMDGNC (c) OMDGPA
(d) OMEFPA (e) None of these
Sol. (d).
The first, third and fifth letters in the word are
moved two, three and four steps forward respectively
while the second, fourth and sixth letters are moved
two, three and four steps backward respectively to
obtain the corresponding letters of the code.
11. In a certain code, POETRY is written as
QONDSQX and OVER is written as PNUDQ.
How is MORE written in that code language ?
(a) LNNQD (b) NNNQD (c) NLNQD
(d) NLPQD (e) None of these
Sol. (c).
The first letter of the word is replaced by a set of two
letters — one following it and the other preceding it
— in the code. The remaining letters of the word are
each moved one step backward to obtain the
remaining letters of the code, in order.
8. In a certain code, BELIEF is written as AFKKDI.
How is SELDOM written in that code ?
(a) RDKCNL (b) RFKENM
(c) RFKFNP (d) TFKENP
(e) None of these
Sol. (c).
The first, third and fifth letters of the word are each
moved one step backward ; the second, fourth and
sixth letters are moved one, two and three steps
forward respectively to obtain the corresponding
letters of the code.
10. If DELHI can be coded as CCIDD, how would you
code BOMBAY ?
(a) AJMTVT (b) AMJXVS
(c) MJXVSU (d) WXYZAX
Sol. (b).
The first, second, third, fourth, ….. letters of the
word are moved one, two, three, four, …. steps
backward respectively to obtain the corresponding
letters of the code.
Slide 4 of 42
13. If POND is coded as RSTL, how is HEAR written
in that code ?
(a) GHIJ (b) GHIZ (c) JIGZ
(d) JCLZ (e) None of these
Sol. (c).
The first, second, third and fourth letters of the word
are moved two, four, six and eight letters forward
respectively to obtain the code.
15. If SYSTEM is coded as SYSMET and NEARER as
AENRER, then FRACTION will be coded as -
(a) CARFNOIT (b) NOITFRAC
(c) FRACNOIT (d) CARFTION
Sol. (a).
The letters in the first half and second half of the
word are written in the reverse order to obtain the
code.
12. If BOMBY is written as MYMYMY, how will
TAMIL NADU be written in that code ?
(a) TIATIATIA (b) MNUMNUMNU
(c) IATIATIAT (d) ALDALDALD
(e) None of these
Sol. (b).
The letters at the third and sixth places are repeated
thrice to code BOMBAY as MYMYMY. Similarly, the
letters at the third, sixth and ninth places are
repeated thrice to code TAMIL NADU as
MNUMNUMNU.
14. In a certain code, INACTIVE is written as
VITCANIE. How is COMPUTER written in the
same code ?
(a) PMOCRETU (b) ETUPMOCR
(c) UTEPMOCR (d) MOCPETUR
Sol. (b).
All the letters of the word, except the last letter, are
written in a reverse order to obtain the code.
Slide 5 of 42
Type - 2
1. If in a certain language, POPULAR is coded as
QPQVMBS, which word would be coded as
GBNPVT ?
(a) FARMER (b) FAMOUS (c) FRAMES
(d) FARMES (e) FAMOTH
Sol. (b).
Each letter of the word is one step behind the
corresponding letter of the code.
2. If in a certain language, UTENSIL is coded as
WVGPUKN, which word would be coded as
DMSFXG ?
(a) BKQEVE (b) BKQDWE (c) BKQDWF
(d) BKQDVE (e) BKQDVF
Sol. (e).
Each letter in the word is two steps behind the
corresponding letter of the code.
3. If ROBUST is coded as QNATRS in a certain
language, which word would be coded as ZXCMP ?
(a) BZEOR (b) AYDNQ (c) AWDLQ
(d) YYBNO (e) YWBLO
Sol. (b).
Each letter in the word is one step ahead of the
corresponding letter of the code.
4. If EHFNRQ is the code for BECKON, which word
has the code QDFWXULQ ?
(a) NCAUTIRN (b) NACUTIRN
(c) NATCRIUN (d) NACTURIN
(e) NACUTRIN
Sol. (d).
Each letter of the word is three steps behind the
corresponding letter of the code.
Slide 6 of 42
6. If in a certain language, GRASP is coded as BMVNK,
which word would be coded As CRANE ?
(a) FUDQH (b) HWFSJ (c) GVERI
(d) XMVIZ (e) BQZMD
Sol. (b).
Each letter of the word is five steps ahead of the
corresponding letter of the code.
8. If in a certain language, TRIANGLE is coded as
SQHZMFKD, which word would be coded as
DWZLOKD.
(a) EXAMPLE (b) FIGMENT (c) DISMISS
(d) DISJOIN (e) None of these
Sol. (a).
Each letter of the word is one step ahead of the
corresponding letter of the code.
5. In a certain code, REFRIGERATOR is coded as
ROTAREGIRFER. Which word would be coded as
NOITINUMMA ?
(a) ANMOMIUTNI (b) AMNTOMUIIN
(c) AMMUNITION (d) NMMUNITIOA
(e) None of these
Sol. (c).
The order of letters of the word is reversed in the
code. So, reverse the order of the letters in the code to
get the word.
7. If in a certain language, COVET is written as
FRYHW, which word would be written as SHDUO ?
(a) QUAKE (b) REPAY (c) STINK
(d) PEARL (e) TIEVP
Sol. (d).
Each letter of the word is three steps behind the
corresponding letter of the code.
Slide 7 of 42
10. If in a certain code, SWITCH is written as TVJSDG,
which word would be written as CQFZE ?
(a) BARED (b) BRAED (c) BREAD
(d) BRADE (e) BRDAE
Sol. (c).
Each letter at odd place in the word is one step
behind and each letter at even place in the word is
one step ahead of the corresponding letter of the
code.
12. If FULFNHW is the code for CRICKET, then
EULGH is the code for which word ?
(a) PRIDE (b) BRIDE (c) BLADE
(d) BLIND (e) None of these
Sol. (b).
Each letter of the word is three steps behind the
corresponding letter of the code.
9. In a code language, SOLID is written as
WPSLPIMFHA. What does the code- word
ATEXXQIBVO refer to ?
(a) WATER (b) WAGER
(c) EAGER (d) WAFER
Sol. (a).
Clearly, the word can be obtained by moving the
letters at even-numbered positions in the code, three
steps forward and omitting those at odd-numbered
positions.
11. If in a certain language, REMOTE is coded
ROTEME, which word would be coded as PNIICC ?
(a) NPIICC (b) PICCIN (c) PINCIC
(d) PICNIC (e) PICINC
Sol. (d).
The groups of second and third letters and fourth
and fifth letters in the word interchange places in the
code.
Slide 8 of 42
14. If LBAEHC is the code for BLEACH, then which of
the following is coded as NBOLZKMH ?
(a) OBNKZLHM (b) LOBNHMKZ
(c) OCPMALNI (d) MANKYJLG
(e) BNLOKZHM
Sol. (e).
The word is divided into groups of two letters each
and then letters in each pair are reversed.
13. If in a certain language, SHIFT is coded as
RFFBO, which word would be coded as LKUMB ?
(a) MMXQG (b) MLVNC (c) KJVLA
(d) MJVLC (e) KJTLA
Sol. (a).
The first, second, third, fourth and fifth letters in the
word are respectively one, two, three, four and five
steps ahead of the corresponding letter of the code.
15. In a certain code, DECEMBER is written as
ERMBCEDE. Which word will be written as
ERMBVENO in that code ?
(a) AUGUST (b) SEPTEMBER
(c) OCTOBER (d) NOVEMBER
Sol. (d).
The word is divided into groups of two letters each
and then these groups are written in the reverse
order.
Slide 9 of 42
Type - 3
Directions :
Below are given letters A to Z. Under each capital letter, a small letter is written which is to be used as a code for the
capital letter.
A B C D E F G H I J K L M N O P Q R S T U V W X Y Z
o z f t g l q n a v s w c b y h u d j r p x m e k i
In each of the following questions, a group of six capital letters is given and its equivalent code is given in one of the
columns (a), (b), (c) or (d). Study the group of letters given in each question and with the help of codes given above,
choose the code equivalent from amongst (a), (b), (c) or (d) as your answer.
(a) (b) (c) (d)
qvscjx ctloig zrmtis qvsliz
tdjiwr wguxzg ataydx nxadjw
afchij gjbflr tdihwr gjbrfl
wguxgz dmthpr wgurvs ataydp
qujoxs ataynp wguxgz tdiwrh
giavyu naxowd grpmil qvpjrp
Slide 10 of 42
1. ESNTCF
Sol. (d).
The code for ESNTCF is gjbrfl.
2. DRZPLT
Sol. (c).
The code for DRZPLT is tdihwr.
3. ICMPZS
Sol. (a).
The code for ICMPZS is afchij.
4. HIVALR
Sol. (b).
The code for HIVALR is naxowd.
5. BTWDZK
Sol. (c).
The code for BTWDZK is zrmtis.
6. IDIORV
Sol. (c).
The code for IDIORV is ataydx.
7. LEQVEB
Sol. (a).
The code for LEQVEB is wguxgz.
8. GJKMSV
Sol. (a).
The code for GJKMSV is qvscjx.
Slide 11 of 42
10. In a certain code, FIRE is written as QHOE and
MOVE as ZMWE. Following the same rule of coding,
what should be the code for the word OVER?
(a) MWED (b) MWEO
(c) MWOE (d) MWZO
Sol. (b).
The code for OVER is MWEO.
12. In a coding system, SHEEP is written as GAXXR
and BLEAT as HPXTN. How can SLATE be written
in this coding system ?
(a) GPTNX (b) GPTXN
(c) GPXNT (d) PTGXN
Sol. (a).
The code for SLATE is GPTNX.
9. If R is denoted by N, D is denoted by T, I by U, O by
I, E by R, T by O, U by D, N by C and C by E, then
how will the word INTRODUCE be written ?
(a) UCONIDTER (b) UCONITDER
(c) UCONTIDER (d) UCOINTDER
(e) None of these
Sol. (b).
11. In a coding system, PEN is written as NZO and BARK
as CTSL. How can we write PRANK in that coding
system ?
(a) CSTZN (b) NSTOL
(c) NTSLO (d) NZTOL
Sol. (b).
The code for PRANK is NSTOL.
I N T R O D U C E
↓ ↓ ↓ ↓ ↓ ↓ ↓ ↓ ↓
U C O N I T D E R
Letter F I R E M O V
Code Q H O E Z M W
Letter P E N B A R K
Code N Z O C T S L
Letter S H E P B L A T
Code G A X R H P T N
Slide 12 of 42
14. In a code language, TUTORIAL is written as
DODNGLCF and DANCE is written as YCJMZ, how
can EDUCATION be written in that code ?
(a) ZYMODCLNJ (b) ZYOMCDLNJ
(c) ZYOMDCLNJ (d) ZYOTNLCMD
(e) None of these
Sol. (b).
The code for EDUCATION is ZYOMCDLNJ.
13. In a particular way of coding, the word CENTRAL
is coded as ABCDEFG and PLANETARIUM as
HGFCBDFEIJK. With the same coding, how can we
express the word LANTERN ?
(a) GFCDFEG (b) GFCDEFG
(c) GFCDBEC (d) GFCDBEB
Sol. (c).
The code for LANTERN is GFCDBEC.
15. If MINERAL is written as QRSTUVW and SOUND is
written as ABCSD, then how will READER be written
in the same code ?
(a) SBFEFS (b) UTVDTU
(c) TUDVUT (d) QDZCDQ
Sol. (b).
The code for READER is UTVDTU.
Letter C E N T R S L P I U M
Code A B C D E F G H I J K
Letter M I N E R A L S O U D
Code Q R S T U V W A B C D
Letter T U O R I A L D N C E
Code D O N G L C F Y J M Z
Slide 13 of 42
Type - 4
1. If REQUEST is written as S2R52TU, then how will
ACID be written ?
(a) 1394 (b) IC94 (c) BDJE
(d) B3J4 (e) None of these
Sol. (e).
Clearly, vowels A, E, I, O, U are coded as 1, 2, 3, 4, 5
respectively. Each of the consonants in the word is
moved one step forward to give the corresponding
letter of the code. So, the code for ACID becomes
1D3E.
2. If each of the letters in the English alphabet is
assigned odd numerical value beginning A = 1,
B = 3 and so on, what will be the total value of the
letters of the word INDIAN ?
(a) 86 (b) 88 (c) 89
(d) 96 (e) None of these
Sol. (d).
Clearly, we have :
So, INDIAN = I + N + D + I + A + N
= 17 + 27 + 7 + 17 + 1 + 27
= 96.
Letter A B C D E F G H I J K L M N
Code 1 3 5 7 9 11 13 15 17 19 21 23 25 27
Slide 14 of 42
3. In a certain code, the word DEAL is coded as 4 −
5 −1 − 12. Following the same rule of coding,
what should be the code for the word LADY ?
(a) 12 − 4 − 1 − 25 (b) 12 − 1 −4 − 25
(c) 10 − 1 − 4 − 23 (d) 12 − 1 −4 − 22
Sol. (b).
Clearly, each letter is coded by the numeral
denoting its position in the English alphabet.
Thus, A is coded as 1, B as 2, C as 3, D as 4, …., L
as 12, M as 13, …., Y as 25, Z as 26.
So, the code for LADY is 12 −1− 4 − 25.
4. If A = 2, M = 26, Z = 52, then BET = ?
(a) 44 (b) 54 (c) 64 (d) 72
Sol. (b).
Clearly, each letter is assigned a numerical value which
is twice the numeral denoting its position in the
English alphabet.
B, E and T and 2nd, and 20th
letters respectively.
So, BET = B + E + T = (2 × 2) + (5 × 2) + (20 × 2) = 54.
5. If A = 26, SUN = 27, then CAT = ?
(a) 24 (b) 27 (c) 57 (d) 58
Sol. (c).
Clearly, each letter is represented by the numeral
denoting its position from the end of the English
alphabet i.e. Z = 1, Y 2, ….., M = 14, …., B = 25, A =
26.
Then, SUN = S + U + N = 8 + 6 + 13 = 27.
So, CAT = C + A + T = 24 + 26 + 7 = 57.
6. If in a certain code, BAT = 23 and CAT = 24, then how
will you code BALL ?
(a) 27 (b) 28 (c) 32 (d) 120
Sol. (a).
C is one step ahead of B and the code for CAT is 1
more than that for BAT. Thus, the letters are coded by
numerals denoting their positions in the English
alphabet.
i.e. A = 1, B = 2, …., Z = 26.
So, BALL = B + A + L + L = 2 + 1 + 12 + 12 = 27.
Slide 15 of 42
7. If GO = 32, SHE = 49, then SOME will be equal to -
(a) 56 (b) 58 (c) 62 (d) 64
Sol. (a).
In the given code, Z = 1, Y = 2, X = 3, …., C = 24,
B = 25, Z = 26.
So, GO = 20 + 12 = 32 and SHE = 8 + 19 + 22 = 49.
Similarly, SOME = S + O + M + E
= 8 + 12 + 14 + 22 = 56.
8. If AT = 20, BAT = 40, then CAT will be equal to -
(a) 30 (b) 50 (c) 60 (d) 70
Sol. (c).
Taking A = 1, B = 2, …., T = 20, …., Z = 26, we have :
AT = A × T = 1 × 20 = 20 ;
BAT = B × A × T = 2 × 1 × 20 = 40.
So, CAT = C × A × T = 3 × 1 × 20 = 60.
9. If ZIP = 198 and ZAP = 246, then how will you code
VIP ?
(a) 174 (b) 222 (c) 888 (d) 990
Sol. (b).
Taking Z = 2, Y = 3, …., N = 14, …., B = 26, A = 27,
we have :
ZIP = (Z + I + P) × 6
= (2 + 19 + 12) × 6 = 33 × 6 = 198.
So, VIP = (V + I + P) × 6
= (6 + 19 + 12) × 6 = 37 × 6 = 222.
10. If DEER = 12215 and HIGH = 5645, how will you
code HEEL ?
(a) 2328 (b) 3449 (c) 4337 (d) 5229
Sol. (d).
Clearly, each letter is coded by a numeral which is 3 less
than the numeral denoting the position of the letter in
the English alphabet. The code for the word is obtained
by joining together physically the number codes for the
individual letters.
H, E, L are 8th, 5th and 12th letters respectively. So,
their codes are 5, 2 and 9 respectively. Thus, the code
for HEEL becomes 5229.
Slide 16 of 42
11. If E = 5 and HOTEL = 12, how will you code LAMB ?
(a) 7 (b) 10 (c) 26 (d) 28
Sol. (a).
We have : A = 1, B = 2, C = 3, …., Y = 25, Z = 26.
Clearly, the code for a word is obtained by dividing the
sum of the individual values of its letters by the
number of letters in the word.
12. If ZEBRA can be written as 2652181, how can
COBRA be written ?
(a) 302181 (b) 3152181
(c) 31822151 (d) 1182153
Sol. (b).
Putting A = 1, B = 2, C = 3, ….., X = 24, Y = 25,
Z = 26, we have :
ZEBRA → Z/E/B/R/A → 26/5/2/18/1 → 2652181.
COBRA → C/O/B/R/A → 3/15/2/18/1 → 3152181.
H + O + T + E + L
Thus, HOTEL =
5
8 15 20 5 12 60
12.
5 5
L + A + M + B 12 1 13 2
So, LAMB =
4 4
28
7.
4
+ + + +
= = =
+ + +
=
= =
Slide 17 of 42
13. If WORK is coded as 4 − 12 − 9 − 16, then how will
you code WOMAN ?
(a) 4 − 12 − 14 − 26 −13 (b) 4 − 26 − 14 − 13 − 12
(c) 23 − 12 − 26 − 14 − 13 (d) 23 − 15 − 13 − 1 − 14
(e) None of these
Sol. (a).
Clearly, each letter is coded by the numeral obtained
by subtracting from 27, the numeral denoting the
position of the letter in the English alphabet. W, O, M,
A, N and 23rd, 15th, 13th, 1st and 14th letters.
So, their codes are (27 − 23), (27 − 15), (27 − 13),
(27 − 1), (27 − 14) i.e. 4, 12, 14, 26, 13 respectively.
14. If ACNE can be coded as 3, 7, 29, 11, then BOIL will
be coded as -
(a) 5, 29, 19, 27 (b) 5, 29, 19, 25
(c) 5, 31, 21, 25 (d) 5, 31, 19, 25
Sol. (d).
In the given coding system, we have :
So, the code for BOIL is 5, 31, 19, 25.
Letter A B C D E F G H I J K L M N O
Code 3 5 7 9 11 13 15 17 19 21 23 25 27 29 31
15. In O = 16, FOR = 42, then what is FRONT equal to ?
(a) 61 (b) 65 (c) 73 (d) 78
Sol. (d).
We have : A = 2m B = 3, …., Z = 27. Then,
FOR = F + O + R = 7 + 16 + 19 = 42.
FRONT = F + R + O +N + T
= 7 + 19 + 16 + 15 + 21 = 78.
Slide 18 of 42
Type - 5
Directions :
The number-group in each question below is to be codified according to the following letter codes :
Number 5 1 3 0 2 4 8 7 6 9
Letter code X L M P D B E F K J
You have to find out which of the answers (a), (b), (c) or (d) has the correct coded form of the given number group.
If none of the coded forms is correct, mark (e) i.e. ‘None of these’ as your answer.
1. 173846
(a) LFMEKB (b) LMFEBK (c) LFMEBK
(d) LFEMBK (e) None of these
Sol. (c).
1 is coded as L, 7 as F, 3 as M, 8 as E, 4 as B, 6 as K.
So, the code for 173846 is LFMEBK.
2. 862941
(a) EDKJBL (b) EKDJLB (c) EKJDBL
(d) EKDJBL (e) None of these
Sol. (d).
8 is coded as E, 6 as K, 2 as D, 9 as J, 4 as B, 1 as L.
So, the code for 862941 is EKDJBL.
Slide 19 of 42
3. 430675
(a) BMKPFX (b) BMPKFX (c) BMPKXF
(d) BMPFKX (e) None of these
Sol. (b).
4 is coded as B, 3 as M, 0 as P, 6 as K, 7 as F, 5 as X.
So, the code for 430675 is BMPKFX.
4. 790853
(a) FJPEXM (b) FPJEXM (c) FJPEMX
(d) FPJEMX (e) None of these
Sol. (a).
7 is coded as F, 9 as J, 0 as P, 8 as E, 5 as X, 3 as M.
So, the code for 790853 is FJPEXM.
5. 901273
(a) JPLDFM (b) JPDLFM (c) JLPDFM
(d) JPLDMF (e) None of these
Sol. (a).
9 is coded as J, 0 as P, 1 as L, 2 as D, 7 as F, 3 as M.
So, the code for 901273 is JPLDFM.
Slide 20 of 42
Directions :
In each of the questions given below, a group of digits followed by four combinations of letter codes labeled (a), (b),
(c) and (d) are given. You have to find out which of the combinations is correct coded form of the group of digits as
per the following codes and conditions. If none of these four combinations in correct, give (e) i.e. ‘None of these’ as
the answer.
Digit 9 3 2 8 1 5 7 6 4 0
Latter code M E B N K R H T D J
6. 328496
(a) YBNDTM (b) XBNDTX (c) EBNTDM
(d) YBNDTY (e) None of these
Sol. (e).
3 is coded as E, 2 as B, 8 as N, 4 as D, 6 as T, 9 as M.
So, the code for 328469 is EBNDTM.
7. 631420
(a) TEKBDW (b) TEKDBW (c) TEKDBJ
(d) TKDEBW (e) None of these
Sol. (b).
Since the last digit is 0, so 0 shall be coded as W. 6 is
coded as T, 3 as E, 1 as K, 4 as D, 2 as B.
So, the code for 631420 is TEKDBW.
Conditions :
(i) If the first digit is an odd number and the last digit is an even number, both are to be coded as ‘X’.
(ii) If the first digit is an even number and the last digit is an odd number, both are to be coded as ‘Y’.
(iii) If either the first or the last digit is 0, then 0 is to be coded as ‘W’.
Slide 21 of 42
8. 640598
(a) TDJRMN (b) TDWRMN (c) XDJRMN
(d) TDJRMY (e) None of these
Sol. (a).
6 is coded as T, 4 as D, 0 as J, 5 as R, 9 as M, 8 as N.
So, the code for 640598 is TDJRMN.
9. 893561
(a) XMERTY (b) NMERTK (c) XMERTX
(d) YMERTY (e) None of these
Sol. (d).
Since the given number-group begins with an even
number and ends with an odd number, so both the
first and last digits i.e. 8 and 1 shall be coded as Y. 9 is
coded as M, 3 as E, 5 as R, 6 as T. So, the code for
893561 is YMERTY.10. 743506
(a) XDERWT (b) HDERWT (c) XDERJX
(d) YDERJY (e) None of these
Sol. (c).
Since the first digit is an odd number and the last
digit is an even number, so both 7 and 6 shall be
coded as X.
4 is coded as D, 3 as E, 5 as R, 0 as J. So, the code for
743506 is XDERJX.
Slide 22 of 42
Directions :
Study the following information carefully to answer these questions :
The digits from 0 to 9 are coded as shown below along with the exceptions I and II :
Digit 3 8 0 7 4 6 9 2 5
Code H $ R A M % L K ξ
11. What will be the code for 764981 ?
(a) A%ML$ξ (b) Y%ML$ξ (c) Y%ML$#
(d) A%ML$# (e) None of these
Sol. (c).
Since the number begins and ends with an odd digit,
so 7 shall be coded as Y and 1 as #. The codes for 6, 4,
9, 8 are %, M, L, $ respectively.
So, the required code is Y%ML$#.
12. What dose MHLEK% represent ?
(a) 439526 (b) 439520(c) 436529
(d) 489526 (e) None of these
Sol. (a).
M, H, L, E, K, % are codes for 4, 3, 9, 5, 2, 6
respectively.
So, MHLEK% represents 439526.
Exception I :
If a number begins and ends with a non-zero odd digit, then the first and the last digits are to be coded as Y and #
respectively.
Exception II :
If a number begins and ends with an even digit (including zero), then the first and the last digits are to be coded as β
and X respectively.
Slide 23 of 42
13. What will be the code for 278140 ?
(a) βA$ξMR (b) βA$ξMX (c) KA$ξMR
(d) YA$ξM# (e) None of these
Sol. (b).
Since 27 8140 has even digits at the first and last
places, so 2 shall be coded as β and 0 as X. The codes
for 7, 8, 1, 4 are A, $, ξ, M respectively.
So, the required code is βA$ξMX
14. What will be the code for 173548 ?
(a) ξAHEK$ (b) ξAREM$ (c) ξAHEM$
(d) ξAHME$ (e) None of these
Sol. (c).
1 is coded as ξ, 7 as A, 3 as H, 5 as E, 4 as M, 8 as $.
So, the code for 1735 48 is ξAHEM$.
15. What does R%LAKξ represent ?
(a) 069725 (b) 697210 (c) 069751
(d) 064721 (e) None of these
Sol. (e).
R, %, L, A, K and ξ are codes for 0, 6, 9, 7, 2 and 1
respectively.
So, R%LAKξ represents 069721.
Slide 24 of 42
Type - 6
Directions :
In each of the following questions, a word is represented by only one set of numbers as given in any one of the
alternatives. The sets of numbers given in the alternatives are represented by two classes of alphabets as in the two
given matrices. The columns and rows of Matrix I are numbers from 0 to 4 and those of Matrix II from 5 to 9. A
letter from these matrices can be represented first by its row and then the column number e.g., in the matrices for
questions 1 to 4, M can be represented by 14, 21, etc ; O can be represented by 20, 32 etc. Similarly you have to
identify the correct set for the word given in each question.
Matrix I
0 1 2 3 4
0 F O M S R
1 S R F O M
2 O M S R F
3 R F O M S
4 M S R F O
Matrix II
5 6 7 8 9
5 A T D I P
6 I P A T D
7 T D I P A
8 P A T D I
9 D I P A T
(Questions)
Slide 25 of 42
1. MOST
(a) 40, 44, 22, 89 (b) 33, 20, 11, 79
(c) 21, 00, 03, 88 (d) 02, 13, 34, 56
Sol. (d).
From matrix I, M can be coded as 02, 14, 21, 33 or 40.
From matrix I, O can be coded as 01, 13, 20, 32 or 44.
From matrix I, S can be coded as 03, 10, 22, 34 or 41.
From matrix II, T can be coded as 56, 68, 75, 87 or 99.
Clearly, (d) is the only set of correct codes.
2. ROAD
(a) 42, 32, 79, 58 (b) 23, 32, 98, 99
(c) 11, 13, 67, 69 (d) 04, 20, 55, 78
Sol. (c).
From matrix I, R can be coded as 04, 11, 23, 30 or 42.
From matrix I, O can be coded as 01, 13, 20, 32 or 44.
From matrix II, A can be coded as 55, 67, 79, 86 or 98.
From matrix II, D can be coded as 57, 69, 76, 88 or 95.
Clearly, (c) contains the correct codes.
3. STOP
(a) 10, 56, 44 , 97 (b) 41, 68, 01, 77
(c) 22, 75, 32, 86 (d) 33, 99, 42, 59
Sol. (a).
From matrix I, S can be coded as 03, 10, 22, 34 or 41.
From matrix II, T can be coded as 56, 68, 75, 87 or 99.
From matrix I, O can be coded as 01, 13, 20, 32 or 44.
From matrix II, P can be coded as 59, 66, 78, 85 or 97.
4. FOAM
(a) 24, 01, 55, 22 (b) 00, 01, 67, 33
(c) 12, 13, 67, 23 (d) 43, 52, 56, 33
Sol. (b).
From matrix I, F can be coded as 00, 12, 24, 31 or 43.
From matrix I, O can be coded as 01, 13, 20, 32 or 44.
From matrix II, A can be coded as 55, 67, 79, 86 or 98.
From matrix I, M can be coded as 02, 14, 21, 33 or 40.
Slide 26 of 42
Matrix I
0 1 2 3 4
0 A E S T H
1 T H A E S
2 E S T H A
3 H A E S T
4 S T H A E
Matrix II
5 6 7 8 9
5 P O R K L
6 K L P O R
7 O R K L P
8 L P O R K
9 R K L P O
(Questions)
5. EAST
(a) 44, 32, 21, 03 (b) 32, 31, 02, 04
(c) 20, 43, 33, 11 (d) 13, 12, 14, 10
Sol. (d).
From matrix I, E can be coded as 01, 13, 20, 32 or 44.
From matrix I, A can be coded as 00, 12, 24, 31 or 43.
From matrix I, S can be coded as 02, 14, 21, 33 or 40.
From matrix I, T can be coded as 03, 10, 22, 34 or 41.
6. ROSE
(a) 95, 75, 02, 32 (b) 88, 76, 31, 32
(c) 86, 67, 33, 44 (d) 57, 87, 32, 33
Sol. (a).
From matrix II, R can be coded as 57, 69, 76, 88 or 95.
From matrix II, O can be coded as 56, 68, 75, 87 or 99.
From matrix I, S can be coded as 02, 14, 21, 33 or 40.
From matrix I, E can be coded as 01, 13, 20, 32 or 44.
Slide 27 of 42
7. SOLE
(a) 41, 57, 87, 31 (b) 33, 99, 66, 44
(c) 21, 75, 44, 02 (d) 02, 78, 87, 13
Sol. (b).
From matrix I, S can be coded as 02, 14, 21, 33 or 40.
From matrix II, O can be coded as 56, 68, 75, 87 or 99.
From matrix II, L can be coded as 59, 66, 78, 85 or 97.
From matrix I, E can be coded as 01, 13, 20, 32 or 44.
8. LAKE
(a) 97, 00, 77, 12 (b) 66, 12, 58, 40
(c) 85, 31, 77, 44 (d) 77, 43, 76, 31
Sol. (c).
From matrix II, L can be coded as 59, 66, 78, 85 or 97.
From matrix I, A can be coded as 00, 12 24, 31 or 43.
From matrix II, K can be coded as 58, 65, 77, 89 or 96.
From matrix I, E can be coded as 01, 13, 20, 32 or 44.
9. LEST
(a) 97, 32, 21, 34 (b) 87, 32, 21, 31
(c) 85, 02, 04, 22 (d) 66, 00, 20, 34
Sol. (a).
From matrix II, L can be coded as 59, 66, 78, 85 or 97.
From matrix I, E can be coded as 01, 13, 20, 32 or 44.
From matrix I, S can be coded as 02, 14, 21, 33 or 40.
From matrix I, T can be coded as 03, 10, 22, 34 or 41.
Slide 28 of 42
Matrix I
0 1 2 3 4
0 F A N O I
1 I O F A N
2 A N O I F
3 O F I N A
4 N I A F O
Matrix II
5 6 7 8 9
5 S E H B T
6 H S E T B
7 B T S E H
8 E H T B S
9 T S E H B
(Questions)
10. NEST
(a) 02, 56, 55, 59 (b) 14, 67, 66, 67
(c) 21, 76, 77, 76 (d) 33, 85, 88, 86
Sol. (a).
From matrix I, N can be coded as 02, 14, 21, 33 or 40.
From matrix II, E can be coded as 56, 67, 78, 85 or 97.
From matrix II, S can be coded as 55, 66, 77, 89 or 96.
From matrix II, T can be coded as 59, 68, 76, 87 or 95.
11. FAITH
(a) 43, 42, 41, 78, 89 (b) 31, 34, 23, 76, 79
(c) 24, 31, 10, 59, 57 (d) 12, 20, 40, 68, 65
Sol. (b).
From matrix I, F can be coded as 00, 12, 24, 31 or 43.
From matrix I, A can be coded as 01, 13, 20, 34 or 42.
From matrix I, I can be coded as 04, 10, 23, 32 or 41.
From matrix II, T can be coded as 59, 68, 76, 87 or 95.
From matrix II, H can be coded as 57, 65, 79, 86 or 98.
Slide 29 of 42
12. FINE
(a) 31, 32, 33, 82 (b) 24, 19, 21, 78
(c) 12, 10, 13, 67 (d) 00, 04, 02, 56
Sol. (d).
From matrix I, F can be coded as 00,12, 24, 31 or 43.
From matrix I, I can be coded as 04, 10, 23, 32 or 41.
From matrix I, N can be coded as 02, 14, 21, 33 or 40.
From matrix II, E can be coded as 56, 67, 78, 85 or 97.
13. HEAT
(a) 79, 53, 20, 87 (b) 65, 56, 13, 57
(c) 57, 56, 01, 59 (d) 29, 85, 34, 93
Sol. (c).
From matrix II, H can be coded as 57, 65, 79, 86 or 98.
From matrix II, E can be coded as 56, 67, 78, 85 or 97.
From matrix I, A can be coded as 01, 13, 20, 34 or 42.
From matrix II, T can be coded as 59, 68, 76, 87 or 95.
14. BOTH
(a) 88, 30, 85, 86 (b) 75, 22, 76, 79
(c) 69, 67, 68, 59 (d) 58, 02, 68, 65
Sol. (b).
From matrix II, B can be coded as 58, 69, 75, 88 or 99.
From matrix I, O can be coded as 03, 11, 22, 30 or 44.
From matrix II, T can be coded as 59, 68, 76, 87 or 95.
From matrix II, H can be coded as 57, 65, 79, 86 or 98.
Slide 30 of 42
Matrix I
0 1 2 3 4
0 D O B A I
1 O B A I D
2 B A I D O
3 A I D O B
4 I D O B A
Matrix II
5 6 7 8 9
5 W N R M L
6 N R M L W
7 R M L W N
8 M L W N R
9 L W N R M
(Questions)
15. DRAW
(a) 41, 66, 23, 55 (b) 32, 75, 44, 76 (c) 23, 57, 30, 68 (d) 14, 89, 12, 78
Sol. (d).
From matrix I, D can be coded as 00, 14, 23, 32 or 41.
From matrix II, R can be coded as 57, 66, 75, 89 or 98.
From matrix I, A can be coded as 03, 12, 21, 30 or 44.
From matrix II, W can be coded as 55, 69, 78, 87 or 96.
Slide 31 of 42
Type - 7
1. If ‘white’ is called ‘blue’, ‘blue’ is called ‘red’, ‘red’ is
called ‘yellow’, ‘yellow’ is called ‘green’, ‘green’ is called
‘black’, ‘black’ is called ‘violet’ and ‘violet’ is called
‘orange’, what would be the colour of human blood ?
Sol. (c).
The colour of the human blood is ‘red’ and as given,
‘red’ is called ‘yellow’. So, the colour of human blood
is ‘yellow’.
3. If ‘pen’ is ‘table’, ‘table’ is ‘fan’, ‘fan’ is ‘chair’ and
‘chair’ is ‘roof’, on which of the following will a
person sit ?
Sol. (c).
A person will sit on a ‘chair’ but a ‘chair’ is called
‘roof’. So, a person will sit on the ‘roof’.
4. If ‘bat’ is ‘racket’, ‘racket’ is ‘football’, ‘football’ is
‘shuttle’, ‘shuttle’ is ‘ludo’ and ‘ludo’ is ‘carrom’,
what is cricket played with ?
Sol. (a).
Cricket is played with a ‘bat’ and ‘bat’ is called
‘racket’. So, cricket is played with a ‘racket’.
(a) Red (b) Green (c) Yellow
(d) Violet (e) Orange
2. If ‘oranges’ are ‘apples’, ‘bananas’ are ‘apricots’,
‘apples’ are ‘chillies’, ‘apricots’ are ‘oranges’, and
‘chillies’ are ‘bananas’, then which of the following are
green in colour ?
Sol. (d).
‘Chillies’ are green in colour and as given, ‘chillies’
are ‘bananas’. So, ‘bananas’ are green in colour.
(a) Apricots (b) Apples (c) Chillies
(d) Bananas (e) Oranges
(a) Fan (b) Chair (c) Roof
(d) Table (e) Pen
(a) Racket (b) Football (c) Bat
(d) Shuttle (e) Carrom
Slide 32 of 42
5. If ‘sky’ is ‘star’, ‘star’ is ‘cloud’, ‘cloud’ is ‘earth’, ‘earth’
is ‘tree’ and ‘tree’ is ‘book’, then where do the birds
fly ?
Sol. (c).
Birds fly in the ‘sky’ and as given, ‘sky’ is ‘star’. So,
birds fly in the ‘star’.
7. If ‘book’ is called ‘watch’, ‘watch’ is called ‘bag’,
‘bag’ is called ‘dictionary’ and ‘dictionary’ is called
‘window’, which is used to carry the books ?
Sol. (a).
Clearly, a ‘bag’ is used to carry the books but a ‘bag’ is
called ‘dictionary’. So, a ‘dictionary’ will be used to
carry the books.
8. If ‘cushion’ is called ‘pillow’, ‘pillow’ is called
‘mat’, ‘mat’ is called ‘bed sheet’ and ‘bed sheet’ is
called ‘cover’, which will be spread on the floor ?
Sol. (b).
‘Mat’ will be spread on the floor. But ‘mat’ is called
‘bed sheet’. So, a man will sleep on the ‘window’.
(a) Cloud (b) Sky (c) Star
(d) Data inadequate (e) None of these
6. If ‘room’ is called ‘bed’, ‘bed’ is called ‘window’,
‘window’ is called ‘flower’ and ‘flower’ is called ‘cooler’,
on what would a man sleep ?
Sol. (a).
A man sleeps on a ‘bed’ and as given, ‘bed’ is called
‘window’. So, a man will sleep on the ‘window’.
(a) Window (b) Bed (c) Flower
(d) Cooler (e) None of these
(a) Dictionary (b) Bag (c) Book
(d) Watch (e) None of these
(a) Cover (b) Bed sheet (c) Mat
(d) Pillow (e) None of these
Slide 33 of 42
9. If ‘orange’ is called ‘butter’, ‘butter’ is called ‘soap’,
‘soap’ is called ‘ink’, ‘ink’ is called ‘honey’ and ‘honey’
is called ‘orange’, which of the following is used for
washing clothes ?
Sol. (e).
Clearly, ‘soap’ is used for washing the clothes. But,
‘soap’ is called ‘ink’. So, ‘ink’ is used for washing the
clothes.
11. If ‘bangle’ is called ‘cassette’, ‘cassette’ is called
‘table’, ‘table’ is called ‘game’ and ‘game’ is called
‘cupboard’, then which is played in the tape
recorder ?
Sol. (c).
Clearly, a ‘cassette’ is played in the tape-recorder. But
a ‘cassette’ is called ‘table’. So, a ‘table’ will be played
in the tape-recorder.
12. If ‘black’ means ‘pink’, ‘pink’ means ‘blue’, ‘blue’
means ‘white’, ‘white’ means ‘yellow’, ‘yellow’
means ‘red’ and ‘red’ means ‘brown’, then what is
the colour of clear sky ?
Sol. (d).
The colour of clear sky is ‘blue’. But, as given, ‘pink’
means ‘blue’. So, the colour of clear sky is ‘pink’.
(a) Honey (b) Butter (c) Orange
(d) Soap (e) Ink
10. If ‘sand’ is called ‘air’, ‘air’ is called ‘plateau’, ‘plateau’
is called ‘well’, ‘well’ is called ‘island’ and ‘island’ is
called ‘sky’, then from where will a woman draw
water ?
Sol. (b).
A woman shall draw water from a ‘well’ but a ‘well’ is
called ‘island’. So, the woman will draw water from
an ‘island’.
(a) Well (b) Island (c) Sky
(d) Air (e) None of these
(a) Bangle (b) Cassette (c) Table
(d) Cupboard (e) None of these (a) Brown (b) Red (c) Blue
(d) Pink (e) None of these
Slide 34 of 42
13. If ‘rain’ is ‘water’, ‘water’ is ‘road’, ‘road’ is ‘cloud’,
‘cloud’ is ‘sky’, ‘sky’ is ‘sea’ and ‘sea’ is ‘path’, where do
aeroplanes fly
Sol. (b).
The aeroplanes fly in the ‘sky’ and the ‘sky’ is called
‘sea’. So, the aeroplanes fly in the ‘sea’.
15. If ‘dust’ is called ‘air’, ‘air’ is called ‘fire’, ‘fire’ is
called ‘water’, ‘water’ is called ‘colour’, ‘colour’ is
called ‘rain’ and ‘rain’ is called ‘dust’, then where do
fish live ?
Sol. (c).
Fishes live in ‘water’ and as given, ‘water’ is called
‘colour’. So, fishes live in ‘colour’.
(a) Road (b) Sea (c) Cloud
(d) Water (e) None of these
14. If ‘water’ is called ‘food’, ‘food’ is called ‘tree’, ‘tree’ is
called ‘sky’, ‘sky’ is called ‘wall’ on which of the
following grows a fruit ?
Sol. (c).
A fruit grows on a ‘tree’ and ‘tree’ is called ‘sky’. So, a
fruit grows on the ‘sky’.
(a) Water (b) Food (c) Sky
(d) Tree (e) Wall
(a) Fire (b) Water
(c) Colour (d) Dust
Slide 35 of 42
Type - 8
1. In a certain code language, ‘col tip mot’ means ‘singing
is appreciable’, ‘mot baj min’ means ‘dancing is good’
and ‘tip nop baj’ means ‘singing and dancing’, which
of the following means ‘good’ in that code language ?
Sol. (b).
In the first and second statements, the common
code-word is ‘mod’ and the common word is ‘is’.
So, ‘mot’ means ‘is’. In the second and third
statements, the common code-word is ‘baj’ and the
common word is ‘dancing’.
So, ‘baj’ means ‘dancing’. Thus, in the second
statement, ‘min’ means ‘good’.
(a) not (b) min (c) baj
(d) Cannot be determined (e) None of these
2. In a certain code language, ‘mink yang pe’ means ‘fruits
are ripe’, ‘pe lao may mink’ means ‘oranges are not ripe’
and ‘may pe nue mink’ means ‘mangoes are not ripe’.
Which word in that language means ‘mangoes’ ?
Sol. (c).
In the second and third statements, the common
code-words are ‘pe’, ‘mink’ and ‘may’ and the
common words are ‘are’, ‘not’ and ‘ripe’.
So, in the third statement, ‘nue’ means ‘mangoes’.
(a) may (b) pe (c) nue (d) mink
Slide 36 of 42
3. In a certain code language, ‘tom kun sud’ means
‘dogs are barking’, ‘kun jo mop’ means ‘dogs and
horses’ and ‘mut tom ko’ means ‘donkeys are mad’.
Which word in that language means ‘barking’ ?
Sol. (a).
In the first and second statements, the common
code-word is ‘kun’ and the common word is ‘dogs’.
So, ‘kun’ means ‘dogs’.
In the first and third statements, the common code-
word is ‘tom’ and the common word is ‘are’.
So, ‘tom’ means ‘are’.
Thus, in the first statement, ‘sud’ means ‘barking’.
(a) sud (b) kun (c) jo
(d) tom (e) ko
4. In a code language, ‘mok dan sil’ means ‘nice big
house’, ‘fit kon dan’ means ‘house is good’ and ‘warm
tir fit’ means ‘cost is hogh’. Which word stands for
‘good’ in that language ?
Sol. (d).
In the first and second statements, the common
code-word is ‘dan’ and the common word is ‘house’.
So, ‘dan’ means ‘house’.
In the second and third statements, the common
code-word is ‘fit’ and the common word is ‘is’.
So, ‘fit’ means ‘is’.
Thus, in the second statement, ‘kon’ stands for ‘good’.
(a) mok (b) dan (c) fit (d) kon
Slide 37 of 42
5. If ‘ski rps tri’ stands for ‘nice Sunday morning’, ‘the
sti rps’ stands for ‘every Tuesday morning’ and ‘ski
ptr qlm’ stands for ‘nice market place’, which word
stands for ‘Sunday’ ?
Sol. (c).
In the first and second statements, the common
code-word is ‘rps’ and the common word is
‘morning’. So, ‘rps’ stands for ‘morning’.
In the first and third statements, the common code-
word is ‘ski’ and the common word is ‘nice’. So, ‘ski’
stands for ‘nice’.
Thus in the second statement, ‘tri’ stands for
‘Sunday’.
(a) vog (b) nat (c) dor
(d) bis (e) None of these
6. In a certain language, ‘pre nat bis’ means ‘smoking is
harmful’, ‘vog dor nat’ means ‘avoid harmful habit’
and ‘dor bis yel’ means ‘please avoid smoking’. Which
of the following means ‘habit’ in that language ?
Sol. (a).
In the first and second statements, the common
code-word is ‘nat’ and the common word is
‘harmful’. So, ‘nat’ means ‘harmful’.
In the second and third statements, the common
code-word is ‘dor ’ and the common word is ‘avoid’.
So, ‘dor’ means ‘avoid’.
Thus, in the second statement, ‘vog’ means ‘habit’.
(a) ski (b) rps (c) tri (d) qlm
Slide 38 of 42
7. In a certain coding system, ‘rbm std bro pus’ means
‘the cat is beautiful’, ‘tnh pus dim std’ means ‘the dog
is brown’, ‘pus dim bro pus cus’ means ‘the dog has
the cat’. What is the code for ‘has’ ?
Sol. (d).
In the third statements, the common code-word ‘pus’
occurs twice and the word ‘the’ also occurs twice.
So, the code-word for ‘the’ is ‘pus’.
Now, in the first and third statements, the common
code-word ‘pus’ stands for ‘the’.
So, the other common code-word ‘bro’ stands for the
other common word i.e. ‘cat’. Similarly, in the second
and third statements, the common code-word ‘dim’
stands for the common word ‘dog’.
Thus, in the third statement, the remaining code-
word i.e. ‘cus’ stands for ‘has’.
(a) lon (b) sig
(c) fin (d) None of these
8. In a certain code language, ‘put tir fin’ means
‘delicious juicy fruit’, ‘tie dip sig’ means ‘beautiful
while lily’ and ‘sig lon fin’ means ‘lily and fruit’.
Which of the following stands for ‘and’ in that
language ?
Sol. (a).
In the first and third statements, the common code-
word i.e. ‘fin’ and the common word is ‘fruit’.
So, ‘fin’ stands for ‘fruit’.
In the second and third statements, the common
code-word is ‘sig’ and the common word is ‘lily’.
So, ‘sig’ stands for ‘lily’.
Thus, in the third statement, ‘lon’ stands for ‘and’.
(a) std (b) dim (c) bro (d) cus
Slide 39 of 42
9. In a certain code language, ‘dom pul ta’ means ‘bring
hot food’, ‘pul tir sop’ means ‘food is good’ and ‘tak
da sop’ means ‘good bright boy’. Which of the
following does mean ‘hot’ in that language ?
Sol. (d).
We can find the code for ‘food’ from the first and
second statements. Now, to find the code for ‘hot’, we
need the code for ‘bring’ which cannot be determined
from the given information.
(a) ja (b) ma (c) kop
(d) top (e) ki
10. In a certain code language, ‘po ki top ma’ means
‘Usha is playing cards’, ‘kop ja ki ma’ means ‘Asha is
playing tennis’, ‘ki top sop ho’ means ‘they are
playing football’ and ‘po sur kop’ means ‘cards and
tennis’. Which word in that language means ‘Asha’ ?
Sol. (a).
In the first and second statements, the common
code-words are ‘ki’ and ‘ma’ and the common words
are ‘is’ and ‘playing’. So, ‘ki’ and ‘ma’ are the codes for
‘is’ and ‘playing’.
In the second and fourth statements, the common
code-word is ‘kop’ and the common word is ‘tennis’.
So, ‘kop’ stands for ‘tennis’.
Thus, in the second statement, ‘ja’ stands for ‘Asha’.
(a) dom (b) pul (c) ta
(d) Cannot be determined (e) None of these
Slide 40 of 42
11. If ‘cinto baoli tsi nzro’ means ‘her village is Sarurpur’,
‘mhi cinto keepi tsi oind’ means ‘her first love is
literature’ and ‘oind geit tsi cinto pki’ means
‘literature collection is her hobby’, which word would
mean ‘literature’ ?
Sol. (c).
In the first and second statements, the common
code-words are ‘cinto’ and ‘tsi’ and the common
words are ‘her’ and ‘is’. So, ‘cinto’ and ‘tsi’ are the
codes for ‘her’ and ‘is’.
In the second and third statements, the common
code-words are ‘cinto’, ‘tsi’ and ‘oind’ and the
common words are ‘her’, ‘is’ and ‘literature’.
Now, ‘cinto’ and ‘tsi’ are codes for ‘her’ and ‘is’. So,
‘oind’ stands for ‘literature’.
(a) cinto (b) baoli (c) oind (d) geit
Slide 41 of 42
Directions :
(Questions)
In a certain code language,
(A) ‘pit dar na’ means ‘you are good’.
(B) ‘dar tok pa’ means ‘good and bad’.
(C) ‘tim na tok’ means ‘they are bad’.
12. In that language, which word stands for ‘they’ ?
Sol. (c).
In statements (A) and (C), the common code-word is ‘na’
and the common word is ‘are’. So, ‘na’ stands for
‘are’.
In statements (B) and (C), the common code-word is ‘tok’
and the common word is ‘bad’. So, ‘tok’ stands for
‘bad’.
Thus, in the statement (C), ‘tim’ stands for ‘they’.
(a) na (b) tok (c) tim (d) pit
(e) None of these
13. To find the answer to the above question, which of
the given statements can be dispensed with ?
Sol. (e).
Since all the given statements have been used to find the
answer, so none of the given statements can be
dispensed with.
(a) Only A (b) Only B (c) A or B (d) B and C
(e) None of these
Slide 42 of 42
Directions :
(Questions)
In a certain code language,
(A) ‘pic vic nic’ means ‘winter is cold.
(B) ‘to nic re’ means ‘summer is hot’.
(C) ‘re pic boo’ means ‘winter and summer’.
(D) ‘vic tho pa’ means ‘nights are cold’.
14. Which word in that language means ‘summer’ ?
Sol. (b).
In statements (B) and (C), the common code-word is ‘re’
and the common word is ‘summer’. So,‘re’ means
‘summer’.
(a) nic (b) re (c) to (d) pic
(e) vic
15. Which of the given statements is superfluous and
can be dispensed with while answering the above
question ?
Sol. (c).
Clearly, both statements (A) and (D) are superfluous.
(a) Only A (b) Only D
(c) Both A and D (d) Neither A nor D
(e) None of these

More Related Content

What's hot

8. Sınıf Kareköklü Sayılar Yaprak Testi 1
8. Sınıf Kareköklü Sayılar Yaprak Testi 18. Sınıf Kareköklü Sayılar Yaprak Testi 1
8. Sınıf Kareköklü Sayılar Yaprak Testi 1sorucanavari
 
Test 1 Ardışık Sayılar
Test 1 Ardışık SayılarTest 1 Ardışık Sayılar
Test 1 Ardışık Sayılarsorucanavari
 
Top down parsing(sid) (1)
Top down parsing(sid) (1)Top down parsing(sid) (1)
Top down parsing(sid) (1)Siddhesh Pange
 
Test 1 Tek ve Çift Sayılar
Test 1 Tek ve Çift SayılarTest 1 Tek ve Çift Sayılar
Test 1 Tek ve Çift Sayılarsorucanavari
 
sequence and series presentatation
sequence and series presentatationsequence and series presentatation
sequence and series presentatationAnishDhakal11
 
Coding Decoding Reasoning Tricks for Competitive Exams
Coding Decoding Reasoning Tricks for Competitive ExamsCoding Decoding Reasoning Tricks for Competitive Exams
Coding Decoding Reasoning Tricks for Competitive ExamsEntrance Exam Info
 
Quick Notes (ENGLISH) For 10th
Quick Notes (ENGLISH) For 10th Quick Notes (ENGLISH) For 10th
Quick Notes (ENGLISH) For 10th Sajjad Ahmed
 
Test 1 - Üslü Sayılar
Test 1 - Üslü SayılarTest 1 - Üslü Sayılar
Test 1 - Üslü Sayılarsorucanavari
 
Test 2 Faktöriyel
Test 2 FaktöriyelTest 2 Faktöriyel
Test 2 Faktöriyelsorucanavari
 
Aptitude Age Problems
Aptitude Age ProblemsAptitude Age Problems
Aptitude Age ProblemsSabith Byari
 
Floyd Warshall Algorithm
Floyd Warshall Algorithm Floyd Warshall Algorithm
Floyd Warshall Algorithm Imamul Kadir
 
Computer hardware concepts mcq paper
Computer hardware concepts mcq paperComputer hardware concepts mcq paper
Computer hardware concepts mcq paperSuresh Khanal
 
9. Sınıf Matematik Deneme Sınavı 1 - Cevap Anahtarlı
9. Sınıf Matematik Deneme Sınavı 1 - Cevap Anahtarlı9. Sınıf Matematik Deneme Sınavı 1 - Cevap Anahtarlı
9. Sınıf Matematik Deneme Sınavı 1 - Cevap Anahtarlısorucanavari
 
CLASS V MATHS
CLASS V MATHSCLASS V MATHS
CLASS V MATHSRc Os
 
NUML Admission Spring 2023.pdf
NUML Admission Spring 2023.pdfNUML Admission Spring 2023.pdf
NUML Admission Spring 2023.pdfAliRaza899305
 
Combined topic 4
Combined topic 4Combined topic 4
Combined topic 4David Ramos
 
CLASS III MATHS SLATE PAPER
CLASS III MATHS SLATE PAPERCLASS III MATHS SLATE PAPER
CLASS III MATHS SLATE PAPERRc Os
 
Quantitative Aptitude Shortcut tricks
Quantitative Aptitude Shortcut tricksQuantitative Aptitude Shortcut tricks
Quantitative Aptitude Shortcut tricksBankExamsToday.com
 

What's hot (20)

Cutting edge starter worksheet
Cutting edge starter worksheetCutting edge starter worksheet
Cutting edge starter worksheet
 
8. Sınıf Kareköklü Sayılar Yaprak Testi 1
8. Sınıf Kareköklü Sayılar Yaprak Testi 18. Sınıf Kareköklü Sayılar Yaprak Testi 1
8. Sınıf Kareköklü Sayılar Yaprak Testi 1
 
Test 1 Ardışık Sayılar
Test 1 Ardışık SayılarTest 1 Ardışık Sayılar
Test 1 Ardışık Sayılar
 
Top down parsing(sid) (1)
Top down parsing(sid) (1)Top down parsing(sid) (1)
Top down parsing(sid) (1)
 
Test 1 Tek ve Çift Sayılar
Test 1 Tek ve Çift SayılarTest 1 Tek ve Çift Sayılar
Test 1 Tek ve Çift Sayılar
 
sequence and series presentatation
sequence and series presentatationsequence and series presentatation
sequence and series presentatation
 
Coding Decoding Reasoning Tricks for Competitive Exams
Coding Decoding Reasoning Tricks for Competitive ExamsCoding Decoding Reasoning Tricks for Competitive Exams
Coding Decoding Reasoning Tricks for Competitive Exams
 
Quick Notes (ENGLISH) For 10th
Quick Notes (ENGLISH) For 10th Quick Notes (ENGLISH) For 10th
Quick Notes (ENGLISH) For 10th
 
Test 1 - Üslü Sayılar
Test 1 - Üslü SayılarTest 1 - Üslü Sayılar
Test 1 - Üslü Sayılar
 
Test 2 Faktöriyel
Test 2 FaktöriyelTest 2 Faktöriyel
Test 2 Faktöriyel
 
Logical reasoning
Logical reasoning Logical reasoning
Logical reasoning
 
Aptitude Age Problems
Aptitude Age ProblemsAptitude Age Problems
Aptitude Age Problems
 
Floyd Warshall Algorithm
Floyd Warshall Algorithm Floyd Warshall Algorithm
Floyd Warshall Algorithm
 
Computer hardware concepts mcq paper
Computer hardware concepts mcq paperComputer hardware concepts mcq paper
Computer hardware concepts mcq paper
 
9. Sınıf Matematik Deneme Sınavı 1 - Cevap Anahtarlı
9. Sınıf Matematik Deneme Sınavı 1 - Cevap Anahtarlı9. Sınıf Matematik Deneme Sınavı 1 - Cevap Anahtarlı
9. Sınıf Matematik Deneme Sınavı 1 - Cevap Anahtarlı
 
CLASS V MATHS
CLASS V MATHSCLASS V MATHS
CLASS V MATHS
 
NUML Admission Spring 2023.pdf
NUML Admission Spring 2023.pdfNUML Admission Spring 2023.pdf
NUML Admission Spring 2023.pdf
 
Combined topic 4
Combined topic 4Combined topic 4
Combined topic 4
 
CLASS III MATHS SLATE PAPER
CLASS III MATHS SLATE PAPERCLASS III MATHS SLATE PAPER
CLASS III MATHS SLATE PAPER
 
Quantitative Aptitude Shortcut tricks
Quantitative Aptitude Shortcut tricksQuantitative Aptitude Shortcut tricks
Quantitative Aptitude Shortcut tricks
 

Similar to Questions on Verbal & Non Verbal Reasoning (Coding decoding)

Learn About Simple Tricks For Coding & Decoding
Learn About Simple Tricks For Coding & DecodingLearn About Simple Tricks For Coding & Decoding
Learn About Simple Tricks For Coding & DecodingMukesh Kumar
 
Coding decoading
Coding decoadingCoding decoading
Coding decoadingAre Lavanya
 
Reasoning practice workbook
Reasoning  practice workbookReasoning  practice workbook
Reasoning practice workbookRahul Tiwari
 
CRO Olympiad Exam Sample Paper for Class 4.
CRO Olympiad Exam Sample Paper for Class 4.CRO Olympiad Exam Sample Paper for Class 4.
CRO Olympiad Exam Sample Paper for Class 4.CREST Olympiads
 
CODING DECODINGdtgffftffyutrhdetugrdt.pptx
CODING DECODINGdtgffftffyutrhdetugrdt.pptxCODING DECODINGdtgffftffyutrhdetugrdt.pptx
CODING DECODINGdtgffftffyutrhdetugrdt.pptxpercepta0
 
Coding and decoding.pptx
Coding and decoding.pptxCoding and decoding.pptx
Coding and decoding.pptxCS50Bootcamp
 
Tnpsc group-4-general-english-answer-key
Tnpsc group-4-general-english-answer-keyTnpsc group-4-general-english-answer-key
Tnpsc group-4-general-english-answer-keyramwhitecity93
 
1 vijaya bank-probationary-officers-exam-reasoning-ability-solved-paper-2008
1 vijaya bank-probationary-officers-exam-reasoning-ability-solved-paper-20081 vijaya bank-probationary-officers-exam-reasoning-ability-solved-paper-2008
1 vijaya bank-probationary-officers-exam-reasoning-ability-solved-paper-2008Kumar Nirmal Prasad
 
1 vijaya bank-probationary-officers-exam-reasoning-ability-solved-paper-2008
1 vijaya bank-probationary-officers-exam-reasoning-ability-solved-paper-20081 vijaya bank-probationary-officers-exam-reasoning-ability-solved-paper-2008
1 vijaya bank-probationary-officers-exam-reasoning-ability-solved-paper-2008Kumar Nirmal Prasad
 
Bank of india probationary
Bank of india probationary Bank of india probationary
Bank of india probationary Babasab Patil
 

Similar to Questions on Verbal & Non Verbal Reasoning (Coding decoding) (13)

lecture-2-coding-decoding (2).ppt
lecture-2-coding-decoding (2).pptlecture-2-coding-decoding (2).ppt
lecture-2-coding-decoding (2).ppt
 
Coding_Decoding.pptx
Coding_Decoding.pptxCoding_Decoding.pptx
Coding_Decoding.pptx
 
Learn About Simple Tricks For Coding & Decoding
Learn About Simple Tricks For Coding & DecodingLearn About Simple Tricks For Coding & Decoding
Learn About Simple Tricks For Coding & Decoding
 
Coding decoading
Coding decoadingCoding decoading
Coding decoading
 
Reasoning practice workbook
Reasoning  practice workbookReasoning  practice workbook
Reasoning practice workbook
 
CRO Olympiad Exam Sample Paper for Class 4.
CRO Olympiad Exam Sample Paper for Class 4.CRO Olympiad Exam Sample Paper for Class 4.
CRO Olympiad Exam Sample Paper for Class 4.
 
CODING DECODINGdtgffftffyutrhdetugrdt.pptx
CODING DECODINGdtgffftffyutrhdetugrdt.pptxCODING DECODINGdtgffftffyutrhdetugrdt.pptx
CODING DECODINGdtgffftffyutrhdetugrdt.pptx
 
Coding and decoding.pptx
Coding and decoding.pptxCoding and decoding.pptx
Coding and decoding.pptx
 
Tnpsc group-4-general-english-answer-key
Tnpsc group-4-general-english-answer-keyTnpsc group-4-general-english-answer-key
Tnpsc group-4-general-english-answer-key
 
bank solved-paers
bank solved-paersbank solved-paers
bank solved-paers
 
1 vijaya bank-probationary-officers-exam-reasoning-ability-solved-paper-2008
1 vijaya bank-probationary-officers-exam-reasoning-ability-solved-paper-20081 vijaya bank-probationary-officers-exam-reasoning-ability-solved-paper-2008
1 vijaya bank-probationary-officers-exam-reasoning-ability-solved-paper-2008
 
1 vijaya bank-probationary-officers-exam-reasoning-ability-solved-paper-2008
1 vijaya bank-probationary-officers-exam-reasoning-ability-solved-paper-20081 vijaya bank-probationary-officers-exam-reasoning-ability-solved-paper-2008
1 vijaya bank-probationary-officers-exam-reasoning-ability-solved-paper-2008
 
Bank of india probationary
Bank of india probationary Bank of india probationary
Bank of india probationary
 

More from LearnPick

How To Make The Most Of Mock Tests
How To Make The Most Of Mock TestsHow To Make The Most Of Mock Tests
How To Make The Most Of Mock TestsLearnPick
 
Career in Travel & Tourism Industry
Career in Travel & Tourism IndustryCareer in Travel & Tourism Industry
Career in Travel & Tourism IndustryLearnPick
 
Benefits of Enrolling in Cooking Classes
Benefits of Enrolling in Cooking ClassesBenefits of Enrolling in Cooking Classes
Benefits of Enrolling in Cooking ClassesLearnPick
 
Incredible World Cup Facts You Probably Don't Know
Incredible World Cup Facts You Probably Don't KnowIncredible World Cup Facts You Probably Don't Know
Incredible World Cup Facts You Probably Don't KnowLearnPick
 
Top Five Tips For ISC Computer Practicals
Top Five Tips For ISC Computer PracticalsTop Five Tips For ISC Computer Practicals
Top Five Tips For ISC Computer PracticalsLearnPick
 
Why Attend A Photography School
Why Attend A Photography SchoolWhy Attend A Photography School
Why Attend A Photography SchoolLearnPick
 
Education System In Thailand
Education System In ThailandEducation System In Thailand
Education System In ThailandLearnPick
 
DIGITAL MARKETING Beginner's workshop
DIGITAL MARKETING Beginner's workshopDIGITAL MARKETING Beginner's workshop
DIGITAL MARKETING Beginner's workshopLearnPick
 
Revolutionization Through 3D Printing
Revolutionization Through 3D PrintingRevolutionization Through 3D Printing
Revolutionization Through 3D PrintingLearnPick
 
The Battle Between ICSE and CBSE
The Battle Between ICSE and CBSEThe Battle Between ICSE and CBSE
The Battle Between ICSE and CBSELearnPick
 
7 unique study hacks to improve your memory
7 unique study hacks to improve your memory7 unique study hacks to improve your memory
7 unique study hacks to improve your memoryLearnPick
 
How To Score Maximum in Biological Diagrams
How To Score Maximum in Biological DiagramsHow To Score Maximum in Biological Diagrams
How To Score Maximum in Biological DiagramsLearnPick
 
A Complete Guide to Manual DSLR Photography
A Complete Guide to Manual DSLR PhotographyA Complete Guide to Manual DSLR Photography
A Complete Guide to Manual DSLR PhotographyLearnPick
 
Dussehra-Diwali Special Preparation Ideas For The Entire Family
Dussehra-Diwali Special Preparation Ideas For The Entire FamilyDussehra-Diwali Special Preparation Ideas For The Entire Family
Dussehra-Diwali Special Preparation Ideas For The Entire FamilyLearnPick
 
Are Your Children Getting Enough of These?
Are Your Children Getting Enough of These?Are Your Children Getting Enough of These?
Are Your Children Getting Enough of These?LearnPick
 
Ways to Ensure Children Do Not Miss Studies During the Monsoons
Ways to Ensure Children Do Not Miss Studies During the MonsoonsWays to Ensure Children Do Not Miss Studies During the Monsoons
Ways to Ensure Children Do Not Miss Studies During the MonsoonsLearnPick
 
What Questions to Ask at a Parents Teachers Conference
What Questions to Ask at a Parents Teachers ConferenceWhat Questions to Ask at a Parents Teachers Conference
What Questions to Ask at a Parents Teachers ConferenceLearnPick
 
8 Tips on How to Help Students with Maths
8 Tips on How to Help Students with Maths8 Tips on How to Help Students with Maths
8 Tips on How to Help Students with MathsLearnPick
 
Why Learn Drawing and Painting?
Why Learn Drawing and Painting?Why Learn Drawing and Painting?
Why Learn Drawing and Painting?LearnPick
 
5 manners kids need to know by the age of 5
5 manners kids need to know by the age of 55 manners kids need to know by the age of 5
5 manners kids need to know by the age of 5LearnPick
 

More from LearnPick (20)

How To Make The Most Of Mock Tests
How To Make The Most Of Mock TestsHow To Make The Most Of Mock Tests
How To Make The Most Of Mock Tests
 
Career in Travel & Tourism Industry
Career in Travel & Tourism IndustryCareer in Travel & Tourism Industry
Career in Travel & Tourism Industry
 
Benefits of Enrolling in Cooking Classes
Benefits of Enrolling in Cooking ClassesBenefits of Enrolling in Cooking Classes
Benefits of Enrolling in Cooking Classes
 
Incredible World Cup Facts You Probably Don't Know
Incredible World Cup Facts You Probably Don't KnowIncredible World Cup Facts You Probably Don't Know
Incredible World Cup Facts You Probably Don't Know
 
Top Five Tips For ISC Computer Practicals
Top Five Tips For ISC Computer PracticalsTop Five Tips For ISC Computer Practicals
Top Five Tips For ISC Computer Practicals
 
Why Attend A Photography School
Why Attend A Photography SchoolWhy Attend A Photography School
Why Attend A Photography School
 
Education System In Thailand
Education System In ThailandEducation System In Thailand
Education System In Thailand
 
DIGITAL MARKETING Beginner's workshop
DIGITAL MARKETING Beginner's workshopDIGITAL MARKETING Beginner's workshop
DIGITAL MARKETING Beginner's workshop
 
Revolutionization Through 3D Printing
Revolutionization Through 3D PrintingRevolutionization Through 3D Printing
Revolutionization Through 3D Printing
 
The Battle Between ICSE and CBSE
The Battle Between ICSE and CBSEThe Battle Between ICSE and CBSE
The Battle Between ICSE and CBSE
 
7 unique study hacks to improve your memory
7 unique study hacks to improve your memory7 unique study hacks to improve your memory
7 unique study hacks to improve your memory
 
How To Score Maximum in Biological Diagrams
How To Score Maximum in Biological DiagramsHow To Score Maximum in Biological Diagrams
How To Score Maximum in Biological Diagrams
 
A Complete Guide to Manual DSLR Photography
A Complete Guide to Manual DSLR PhotographyA Complete Guide to Manual DSLR Photography
A Complete Guide to Manual DSLR Photography
 
Dussehra-Diwali Special Preparation Ideas For The Entire Family
Dussehra-Diwali Special Preparation Ideas For The Entire FamilyDussehra-Diwali Special Preparation Ideas For The Entire Family
Dussehra-Diwali Special Preparation Ideas For The Entire Family
 
Are Your Children Getting Enough of These?
Are Your Children Getting Enough of These?Are Your Children Getting Enough of These?
Are Your Children Getting Enough of These?
 
Ways to Ensure Children Do Not Miss Studies During the Monsoons
Ways to Ensure Children Do Not Miss Studies During the MonsoonsWays to Ensure Children Do Not Miss Studies During the Monsoons
Ways to Ensure Children Do Not Miss Studies During the Monsoons
 
What Questions to Ask at a Parents Teachers Conference
What Questions to Ask at a Parents Teachers ConferenceWhat Questions to Ask at a Parents Teachers Conference
What Questions to Ask at a Parents Teachers Conference
 
8 Tips on How to Help Students with Maths
8 Tips on How to Help Students with Maths8 Tips on How to Help Students with Maths
8 Tips on How to Help Students with Maths
 
Why Learn Drawing and Painting?
Why Learn Drawing and Painting?Why Learn Drawing and Painting?
Why Learn Drawing and Painting?
 
5 manners kids need to know by the age of 5
5 manners kids need to know by the age of 55 manners kids need to know by the age of 5
5 manners kids need to know by the age of 5
 

Recently uploaded

Organic Name Reactions for the students and aspirants of Chemistry12th.pptx
Organic Name Reactions  for the students and aspirants of Chemistry12th.pptxOrganic Name Reactions  for the students and aspirants of Chemistry12th.pptx
Organic Name Reactions for the students and aspirants of Chemistry12th.pptxVS Mahajan Coaching Centre
 
JAPAN: ORGANISATION OF PMDA, PHARMACEUTICAL LAWS & REGULATIONS, TYPES OF REGI...
JAPAN: ORGANISATION OF PMDA, PHARMACEUTICAL LAWS & REGULATIONS, TYPES OF REGI...JAPAN: ORGANISATION OF PMDA, PHARMACEUTICAL LAWS & REGULATIONS, TYPES OF REGI...
JAPAN: ORGANISATION OF PMDA, PHARMACEUTICAL LAWS & REGULATIONS, TYPES OF REGI...anjaliyadav012327
 
BASLIQ CURRENT LOOKBOOK LOOKBOOK(1) (1).pdf
BASLIQ CURRENT LOOKBOOK  LOOKBOOK(1) (1).pdfBASLIQ CURRENT LOOKBOOK  LOOKBOOK(1) (1).pdf
BASLIQ CURRENT LOOKBOOK LOOKBOOK(1) (1).pdfSoniaTolstoy
 
Paris 2024 Olympic Geographies - an activity
Paris 2024 Olympic Geographies - an activityParis 2024 Olympic Geographies - an activity
Paris 2024 Olympic Geographies - an activityGeoBlogs
 
A Critique of the Proposed National Education Policy Reform
A Critique of the Proposed National Education Policy ReformA Critique of the Proposed National Education Policy Reform
A Critique of the Proposed National Education Policy ReformChameera Dedduwage
 
Q4-W6-Restating Informational Text Grade 3
Q4-W6-Restating Informational Text Grade 3Q4-W6-Restating Informational Text Grade 3
Q4-W6-Restating Informational Text Grade 3JemimahLaneBuaron
 
Russian Call Girls in Andheri Airport Mumbai WhatsApp 9167673311 💞 Full Nigh...
Russian Call Girls in Andheri Airport Mumbai WhatsApp  9167673311 💞 Full Nigh...Russian Call Girls in Andheri Airport Mumbai WhatsApp  9167673311 💞 Full Nigh...
Russian Call Girls in Andheri Airport Mumbai WhatsApp 9167673311 💞 Full Nigh...Pooja Nehwal
 
Activity 01 - Artificial Culture (1).pdf
Activity 01 - Artificial Culture (1).pdfActivity 01 - Artificial Culture (1).pdf
Activity 01 - Artificial Culture (1).pdfciinovamais
 
The basics of sentences session 2pptx copy.pptx
The basics of sentences session 2pptx copy.pptxThe basics of sentences session 2pptx copy.pptx
The basics of sentences session 2pptx copy.pptxheathfieldcps1
 
Mastering the Unannounced Regulatory Inspection
Mastering the Unannounced Regulatory InspectionMastering the Unannounced Regulatory Inspection
Mastering the Unannounced Regulatory InspectionSafetyChain Software
 
POINT- BIOCHEMISTRY SEM 2 ENZYMES UNIT 5.pptx
POINT- BIOCHEMISTRY SEM 2 ENZYMES UNIT 5.pptxPOINT- BIOCHEMISTRY SEM 2 ENZYMES UNIT 5.pptx
POINT- BIOCHEMISTRY SEM 2 ENZYMES UNIT 5.pptxSayali Powar
 
Student login on Anyboli platform.helpin
Student login on Anyboli platform.helpinStudent login on Anyboli platform.helpin
Student login on Anyboli platform.helpinRaunakKeshri1
 
microwave assisted reaction. General introduction
microwave assisted reaction. General introductionmicrowave assisted reaction. General introduction
microwave assisted reaction. General introductionMaksud Ahmed
 
social pharmacy d-pharm 1st year by Pragati K. Mahajan
social pharmacy d-pharm 1st year by Pragati K. Mahajansocial pharmacy d-pharm 1st year by Pragati K. Mahajan
social pharmacy d-pharm 1st year by Pragati K. Mahajanpragatimahajan3
 
Nutritional Needs Presentation - HLTH 104
Nutritional Needs Presentation - HLTH 104Nutritional Needs Presentation - HLTH 104
Nutritional Needs Presentation - HLTH 104misteraugie
 
SOCIAL AND HISTORICAL CONTEXT - LFTVD.pptx
SOCIAL AND HISTORICAL CONTEXT - LFTVD.pptxSOCIAL AND HISTORICAL CONTEXT - LFTVD.pptx
SOCIAL AND HISTORICAL CONTEXT - LFTVD.pptxiammrhaywood
 
Sports & Fitness Value Added Course FY..
Sports & Fitness Value Added Course FY..Sports & Fitness Value Added Course FY..
Sports & Fitness Value Added Course FY..Disha Kariya
 
Introduction to Nonprofit Accounting: The Basics
Introduction to Nonprofit Accounting: The BasicsIntroduction to Nonprofit Accounting: The Basics
Introduction to Nonprofit Accounting: The BasicsTechSoup
 

Recently uploaded (20)

Organic Name Reactions for the students and aspirants of Chemistry12th.pptx
Organic Name Reactions  for the students and aspirants of Chemistry12th.pptxOrganic Name Reactions  for the students and aspirants of Chemistry12th.pptx
Organic Name Reactions for the students and aspirants of Chemistry12th.pptx
 
JAPAN: ORGANISATION OF PMDA, PHARMACEUTICAL LAWS & REGULATIONS, TYPES OF REGI...
JAPAN: ORGANISATION OF PMDA, PHARMACEUTICAL LAWS & REGULATIONS, TYPES OF REGI...JAPAN: ORGANISATION OF PMDA, PHARMACEUTICAL LAWS & REGULATIONS, TYPES OF REGI...
JAPAN: ORGANISATION OF PMDA, PHARMACEUTICAL LAWS & REGULATIONS, TYPES OF REGI...
 
BASLIQ CURRENT LOOKBOOK LOOKBOOK(1) (1).pdf
BASLIQ CURRENT LOOKBOOK  LOOKBOOK(1) (1).pdfBASLIQ CURRENT LOOKBOOK  LOOKBOOK(1) (1).pdf
BASLIQ CURRENT LOOKBOOK LOOKBOOK(1) (1).pdf
 
Paris 2024 Olympic Geographies - an activity
Paris 2024 Olympic Geographies - an activityParis 2024 Olympic Geographies - an activity
Paris 2024 Olympic Geographies - an activity
 
Advance Mobile Application Development class 07
Advance Mobile Application Development class 07Advance Mobile Application Development class 07
Advance Mobile Application Development class 07
 
A Critique of the Proposed National Education Policy Reform
A Critique of the Proposed National Education Policy ReformA Critique of the Proposed National Education Policy Reform
A Critique of the Proposed National Education Policy Reform
 
INDIA QUIZ 2024 RLAC DELHI UNIVERSITY.pptx
INDIA QUIZ 2024 RLAC DELHI UNIVERSITY.pptxINDIA QUIZ 2024 RLAC DELHI UNIVERSITY.pptx
INDIA QUIZ 2024 RLAC DELHI UNIVERSITY.pptx
 
Q4-W6-Restating Informational Text Grade 3
Q4-W6-Restating Informational Text Grade 3Q4-W6-Restating Informational Text Grade 3
Q4-W6-Restating Informational Text Grade 3
 
Russian Call Girls in Andheri Airport Mumbai WhatsApp 9167673311 💞 Full Nigh...
Russian Call Girls in Andheri Airport Mumbai WhatsApp  9167673311 💞 Full Nigh...Russian Call Girls in Andheri Airport Mumbai WhatsApp  9167673311 💞 Full Nigh...
Russian Call Girls in Andheri Airport Mumbai WhatsApp 9167673311 💞 Full Nigh...
 
Activity 01 - Artificial Culture (1).pdf
Activity 01 - Artificial Culture (1).pdfActivity 01 - Artificial Culture (1).pdf
Activity 01 - Artificial Culture (1).pdf
 
The basics of sentences session 2pptx copy.pptx
The basics of sentences session 2pptx copy.pptxThe basics of sentences session 2pptx copy.pptx
The basics of sentences session 2pptx copy.pptx
 
Mastering the Unannounced Regulatory Inspection
Mastering the Unannounced Regulatory InspectionMastering the Unannounced Regulatory Inspection
Mastering the Unannounced Regulatory Inspection
 
POINT- BIOCHEMISTRY SEM 2 ENZYMES UNIT 5.pptx
POINT- BIOCHEMISTRY SEM 2 ENZYMES UNIT 5.pptxPOINT- BIOCHEMISTRY SEM 2 ENZYMES UNIT 5.pptx
POINT- BIOCHEMISTRY SEM 2 ENZYMES UNIT 5.pptx
 
Student login on Anyboli platform.helpin
Student login on Anyboli platform.helpinStudent login on Anyboli platform.helpin
Student login on Anyboli platform.helpin
 
microwave assisted reaction. General introduction
microwave assisted reaction. General introductionmicrowave assisted reaction. General introduction
microwave assisted reaction. General introduction
 
social pharmacy d-pharm 1st year by Pragati K. Mahajan
social pharmacy d-pharm 1st year by Pragati K. Mahajansocial pharmacy d-pharm 1st year by Pragati K. Mahajan
social pharmacy d-pharm 1st year by Pragati K. Mahajan
 
Nutritional Needs Presentation - HLTH 104
Nutritional Needs Presentation - HLTH 104Nutritional Needs Presentation - HLTH 104
Nutritional Needs Presentation - HLTH 104
 
SOCIAL AND HISTORICAL CONTEXT - LFTVD.pptx
SOCIAL AND HISTORICAL CONTEXT - LFTVD.pptxSOCIAL AND HISTORICAL CONTEXT - LFTVD.pptx
SOCIAL AND HISTORICAL CONTEXT - LFTVD.pptx
 
Sports & Fitness Value Added Course FY..
Sports & Fitness Value Added Course FY..Sports & Fitness Value Added Course FY..
Sports & Fitness Value Added Course FY..
 
Introduction to Nonprofit Accounting: The Basics
Introduction to Nonprofit Accounting: The BasicsIntroduction to Nonprofit Accounting: The Basics
Introduction to Nonprofit Accounting: The Basics
 

Questions on Verbal & Non Verbal Reasoning (Coding decoding)

  • 1. Slide 1 of 42 Type - 1 1. If in a certain code, LUTE is written as MUTE and FATE is written as GATE, then how will BLUE be written in that code ? (a) CLUE (b) GLUE (c) FLUE (d) SLUE Sol. (a). The first letter of the word is moved one step forward to obtain the first letter of the code, while the other letters remain unaltered. 2. If FISH is written as EHRG in a certain code, how would JUNGLE be written in that code ? (a) ITMFKD (b) ITNFKD (c) KVOHMF (d) TIMFKD Sol. (a). Each letter in the word is moved one step backward to obtain the corresponding letter of the code. 3. In a certain code, ROAD is written as URDG. How is SWAN written in that code ? (a) VXDQ (b) VZDQ (c) VZCP (d) UXDQ Sol. (b). Each letter in the word is moved three steps forward to obtain the corresponding letter of the code. 4. In a certain code, FAVOUR is written as EBUPTS. How is DANGER written in that code ? (a) CBFFDS (b) CBMHDS (c) EBFHDS (d) EBHHFS Sol. (b). The first, third and fifth letters are each moved one step backward, while the second, fourth and sixth letters are each moved one step forward to obtain the corresponding letters of the code.
  • 2. Slide 2 of 42 5. In a certain code, PRODUCTIONS id written as QQPCVEUHPMT. How is ORIENTATION written in that code ? (a) PQJDOVBSJNO (b) PQJDOUBUJPO (c) PSJFOVBSJNO (d) NSHFMVBSJNO (e) None of these Sol. (a). The first, third, fifth, seventh, ninth and eleventh letters in the word are each moved one step forward ; the second, fourth, eight and tenth letters are each moved one step backward, while the middle (i.e. sixth) letter is moved two steps forward to obtain the corresponding letters of the code. 6. In a certain code, BASIC is written as DDULE. How is LEADER written in that code ? (a) NGCFGT (b) NHCGGU (c) OGDFHT (d) OHDGHU Sol. (b). The letters at the odd-numbered positions in the word are each moved two steps forward while those at the even-numbered positions are each moved three steps forward to obtain the corresponding letters of the code. 7. If in a certain language, MIRACLE is coded as NKUEHRL, then how is GAMVLE coded in that language ? (a) JDOCMF (b) CLEMNK (c) HCPFQK (d) AELGMN Sol. (c). The first, second, third, fourth, fifth, sixth and seventh letters in the word are moved one, two, three, four, five, six and seven steps forward respectively to obtain the corresponding letters of the code.
  • 3. Slide 3 of 42 9. In a certain code, SPRING is written as UNUFRC. How will the word MOBILE be written in that code language ? (a) KQEFPA (b) OMDGNC (c) OMDGPA (d) OMEFPA (e) None of these Sol. (d). The first, third and fifth letters in the word are moved two, three and four steps forward respectively while the second, fourth and sixth letters are moved two, three and four steps backward respectively to obtain the corresponding letters of the code. 11. In a certain code, POETRY is written as QONDSQX and OVER is written as PNUDQ. How is MORE written in that code language ? (a) LNNQD (b) NNNQD (c) NLNQD (d) NLPQD (e) None of these Sol. (c). The first letter of the word is replaced by a set of two letters — one following it and the other preceding it — in the code. The remaining letters of the word are each moved one step backward to obtain the remaining letters of the code, in order. 8. In a certain code, BELIEF is written as AFKKDI. How is SELDOM written in that code ? (a) RDKCNL (b) RFKENM (c) RFKFNP (d) TFKENP (e) None of these Sol. (c). The first, third and fifth letters of the word are each moved one step backward ; the second, fourth and sixth letters are moved one, two and three steps forward respectively to obtain the corresponding letters of the code. 10. If DELHI can be coded as CCIDD, how would you code BOMBAY ? (a) AJMTVT (b) AMJXVS (c) MJXVSU (d) WXYZAX Sol. (b). The first, second, third, fourth, ….. letters of the word are moved one, two, three, four, …. steps backward respectively to obtain the corresponding letters of the code.
  • 4. Slide 4 of 42 13. If POND is coded as RSTL, how is HEAR written in that code ? (a) GHIJ (b) GHIZ (c) JIGZ (d) JCLZ (e) None of these Sol. (c). The first, second, third and fourth letters of the word are moved two, four, six and eight letters forward respectively to obtain the code. 15. If SYSTEM is coded as SYSMET and NEARER as AENRER, then FRACTION will be coded as - (a) CARFNOIT (b) NOITFRAC (c) FRACNOIT (d) CARFTION Sol. (a). The letters in the first half and second half of the word are written in the reverse order to obtain the code. 12. If BOMBY is written as MYMYMY, how will TAMIL NADU be written in that code ? (a) TIATIATIA (b) MNUMNUMNU (c) IATIATIAT (d) ALDALDALD (e) None of these Sol. (b). The letters at the third and sixth places are repeated thrice to code BOMBAY as MYMYMY. Similarly, the letters at the third, sixth and ninth places are repeated thrice to code TAMIL NADU as MNUMNUMNU. 14. In a certain code, INACTIVE is written as VITCANIE. How is COMPUTER written in the same code ? (a) PMOCRETU (b) ETUPMOCR (c) UTEPMOCR (d) MOCPETUR Sol. (b). All the letters of the word, except the last letter, are written in a reverse order to obtain the code.
  • 5. Slide 5 of 42 Type - 2 1. If in a certain language, POPULAR is coded as QPQVMBS, which word would be coded as GBNPVT ? (a) FARMER (b) FAMOUS (c) FRAMES (d) FARMES (e) FAMOTH Sol. (b). Each letter of the word is one step behind the corresponding letter of the code. 2. If in a certain language, UTENSIL is coded as WVGPUKN, which word would be coded as DMSFXG ? (a) BKQEVE (b) BKQDWE (c) BKQDWF (d) BKQDVE (e) BKQDVF Sol. (e). Each letter in the word is two steps behind the corresponding letter of the code. 3. If ROBUST is coded as QNATRS in a certain language, which word would be coded as ZXCMP ? (a) BZEOR (b) AYDNQ (c) AWDLQ (d) YYBNO (e) YWBLO Sol. (b). Each letter in the word is one step ahead of the corresponding letter of the code. 4. If EHFNRQ is the code for BECKON, which word has the code QDFWXULQ ? (a) NCAUTIRN (b) NACUTIRN (c) NATCRIUN (d) NACTURIN (e) NACUTRIN Sol. (d). Each letter of the word is three steps behind the corresponding letter of the code.
  • 6. Slide 6 of 42 6. If in a certain language, GRASP is coded as BMVNK, which word would be coded As CRANE ? (a) FUDQH (b) HWFSJ (c) GVERI (d) XMVIZ (e) BQZMD Sol. (b). Each letter of the word is five steps ahead of the corresponding letter of the code. 8. If in a certain language, TRIANGLE is coded as SQHZMFKD, which word would be coded as DWZLOKD. (a) EXAMPLE (b) FIGMENT (c) DISMISS (d) DISJOIN (e) None of these Sol. (a). Each letter of the word is one step ahead of the corresponding letter of the code. 5. In a certain code, REFRIGERATOR is coded as ROTAREGIRFER. Which word would be coded as NOITINUMMA ? (a) ANMOMIUTNI (b) AMNTOMUIIN (c) AMMUNITION (d) NMMUNITIOA (e) None of these Sol. (c). The order of letters of the word is reversed in the code. So, reverse the order of the letters in the code to get the word. 7. If in a certain language, COVET is written as FRYHW, which word would be written as SHDUO ? (a) QUAKE (b) REPAY (c) STINK (d) PEARL (e) TIEVP Sol. (d). Each letter of the word is three steps behind the corresponding letter of the code.
  • 7. Slide 7 of 42 10. If in a certain code, SWITCH is written as TVJSDG, which word would be written as CQFZE ? (a) BARED (b) BRAED (c) BREAD (d) BRADE (e) BRDAE Sol. (c). Each letter at odd place in the word is one step behind and each letter at even place in the word is one step ahead of the corresponding letter of the code. 12. If FULFNHW is the code for CRICKET, then EULGH is the code for which word ? (a) PRIDE (b) BRIDE (c) BLADE (d) BLIND (e) None of these Sol. (b). Each letter of the word is three steps behind the corresponding letter of the code. 9. In a code language, SOLID is written as WPSLPIMFHA. What does the code- word ATEXXQIBVO refer to ? (a) WATER (b) WAGER (c) EAGER (d) WAFER Sol. (a). Clearly, the word can be obtained by moving the letters at even-numbered positions in the code, three steps forward and omitting those at odd-numbered positions. 11. If in a certain language, REMOTE is coded ROTEME, which word would be coded as PNIICC ? (a) NPIICC (b) PICCIN (c) PINCIC (d) PICNIC (e) PICINC Sol. (d). The groups of second and third letters and fourth and fifth letters in the word interchange places in the code.
  • 8. Slide 8 of 42 14. If LBAEHC is the code for BLEACH, then which of the following is coded as NBOLZKMH ? (a) OBNKZLHM (b) LOBNHMKZ (c) OCPMALNI (d) MANKYJLG (e) BNLOKZHM Sol. (e). The word is divided into groups of two letters each and then letters in each pair are reversed. 13. If in a certain language, SHIFT is coded as RFFBO, which word would be coded as LKUMB ? (a) MMXQG (b) MLVNC (c) KJVLA (d) MJVLC (e) KJTLA Sol. (a). The first, second, third, fourth and fifth letters in the word are respectively one, two, three, four and five steps ahead of the corresponding letter of the code. 15. In a certain code, DECEMBER is written as ERMBCEDE. Which word will be written as ERMBVENO in that code ? (a) AUGUST (b) SEPTEMBER (c) OCTOBER (d) NOVEMBER Sol. (d). The word is divided into groups of two letters each and then these groups are written in the reverse order.
  • 9. Slide 9 of 42 Type - 3 Directions : Below are given letters A to Z. Under each capital letter, a small letter is written which is to be used as a code for the capital letter. A B C D E F G H I J K L M N O P Q R S T U V W X Y Z o z f t g l q n a v s w c b y h u d j r p x m e k i In each of the following questions, a group of six capital letters is given and its equivalent code is given in one of the columns (a), (b), (c) or (d). Study the group of letters given in each question and with the help of codes given above, choose the code equivalent from amongst (a), (b), (c) or (d) as your answer. (a) (b) (c) (d) qvscjx ctloig zrmtis qvsliz tdjiwr wguxzg ataydx nxadjw afchij gjbflr tdihwr gjbrfl wguxgz dmthpr wgurvs ataydp qujoxs ataynp wguxgz tdiwrh giavyu naxowd grpmil qvpjrp
  • 10. Slide 10 of 42 1. ESNTCF Sol. (d). The code for ESNTCF is gjbrfl. 2. DRZPLT Sol. (c). The code for DRZPLT is tdihwr. 3. ICMPZS Sol. (a). The code for ICMPZS is afchij. 4. HIVALR Sol. (b). The code for HIVALR is naxowd. 5. BTWDZK Sol. (c). The code for BTWDZK is zrmtis. 6. IDIORV Sol. (c). The code for IDIORV is ataydx. 7. LEQVEB Sol. (a). The code for LEQVEB is wguxgz. 8. GJKMSV Sol. (a). The code for GJKMSV is qvscjx.
  • 11. Slide 11 of 42 10. In a certain code, FIRE is written as QHOE and MOVE as ZMWE. Following the same rule of coding, what should be the code for the word OVER? (a) MWED (b) MWEO (c) MWOE (d) MWZO Sol. (b). The code for OVER is MWEO. 12. In a coding system, SHEEP is written as GAXXR and BLEAT as HPXTN. How can SLATE be written in this coding system ? (a) GPTNX (b) GPTXN (c) GPXNT (d) PTGXN Sol. (a). The code for SLATE is GPTNX. 9. If R is denoted by N, D is denoted by T, I by U, O by I, E by R, T by O, U by D, N by C and C by E, then how will the word INTRODUCE be written ? (a) UCONIDTER (b) UCONITDER (c) UCONTIDER (d) UCOINTDER (e) None of these Sol. (b). 11. In a coding system, PEN is written as NZO and BARK as CTSL. How can we write PRANK in that coding system ? (a) CSTZN (b) NSTOL (c) NTSLO (d) NZTOL Sol. (b). The code for PRANK is NSTOL. I N T R O D U C E ↓ ↓ ↓ ↓ ↓ ↓ ↓ ↓ ↓ U C O N I T D E R Letter F I R E M O V Code Q H O E Z M W Letter P E N B A R K Code N Z O C T S L Letter S H E P B L A T Code G A X R H P T N
  • 12. Slide 12 of 42 14. In a code language, TUTORIAL is written as DODNGLCF and DANCE is written as YCJMZ, how can EDUCATION be written in that code ? (a) ZYMODCLNJ (b) ZYOMCDLNJ (c) ZYOMDCLNJ (d) ZYOTNLCMD (e) None of these Sol. (b). The code for EDUCATION is ZYOMCDLNJ. 13. In a particular way of coding, the word CENTRAL is coded as ABCDEFG and PLANETARIUM as HGFCBDFEIJK. With the same coding, how can we express the word LANTERN ? (a) GFCDFEG (b) GFCDEFG (c) GFCDBEC (d) GFCDBEB Sol. (c). The code for LANTERN is GFCDBEC. 15. If MINERAL is written as QRSTUVW and SOUND is written as ABCSD, then how will READER be written in the same code ? (a) SBFEFS (b) UTVDTU (c) TUDVUT (d) QDZCDQ Sol. (b). The code for READER is UTVDTU. Letter C E N T R S L P I U M Code A B C D E F G H I J K Letter M I N E R A L S O U D Code Q R S T U V W A B C D Letter T U O R I A L D N C E Code D O N G L C F Y J M Z
  • 13. Slide 13 of 42 Type - 4 1. If REQUEST is written as S2R52TU, then how will ACID be written ? (a) 1394 (b) IC94 (c) BDJE (d) B3J4 (e) None of these Sol. (e). Clearly, vowels A, E, I, O, U are coded as 1, 2, 3, 4, 5 respectively. Each of the consonants in the word is moved one step forward to give the corresponding letter of the code. So, the code for ACID becomes 1D3E. 2. If each of the letters in the English alphabet is assigned odd numerical value beginning A = 1, B = 3 and so on, what will be the total value of the letters of the word INDIAN ? (a) 86 (b) 88 (c) 89 (d) 96 (e) None of these Sol. (d). Clearly, we have : So, INDIAN = I + N + D + I + A + N = 17 + 27 + 7 + 17 + 1 + 27 = 96. Letter A B C D E F G H I J K L M N Code 1 3 5 7 9 11 13 15 17 19 21 23 25 27
  • 14. Slide 14 of 42 3. In a certain code, the word DEAL is coded as 4 − 5 −1 − 12. Following the same rule of coding, what should be the code for the word LADY ? (a) 12 − 4 − 1 − 25 (b) 12 − 1 −4 − 25 (c) 10 − 1 − 4 − 23 (d) 12 − 1 −4 − 22 Sol. (b). Clearly, each letter is coded by the numeral denoting its position in the English alphabet. Thus, A is coded as 1, B as 2, C as 3, D as 4, …., L as 12, M as 13, …., Y as 25, Z as 26. So, the code for LADY is 12 −1− 4 − 25. 4. If A = 2, M = 26, Z = 52, then BET = ? (a) 44 (b) 54 (c) 64 (d) 72 Sol. (b). Clearly, each letter is assigned a numerical value which is twice the numeral denoting its position in the English alphabet. B, E and T and 2nd, and 20th letters respectively. So, BET = B + E + T = (2 × 2) + (5 × 2) + (20 × 2) = 54. 5. If A = 26, SUN = 27, then CAT = ? (a) 24 (b) 27 (c) 57 (d) 58 Sol. (c). Clearly, each letter is represented by the numeral denoting its position from the end of the English alphabet i.e. Z = 1, Y 2, ….., M = 14, …., B = 25, A = 26. Then, SUN = S + U + N = 8 + 6 + 13 = 27. So, CAT = C + A + T = 24 + 26 + 7 = 57. 6. If in a certain code, BAT = 23 and CAT = 24, then how will you code BALL ? (a) 27 (b) 28 (c) 32 (d) 120 Sol. (a). C is one step ahead of B and the code for CAT is 1 more than that for BAT. Thus, the letters are coded by numerals denoting their positions in the English alphabet. i.e. A = 1, B = 2, …., Z = 26. So, BALL = B + A + L + L = 2 + 1 + 12 + 12 = 27.
  • 15. Slide 15 of 42 7. If GO = 32, SHE = 49, then SOME will be equal to - (a) 56 (b) 58 (c) 62 (d) 64 Sol. (a). In the given code, Z = 1, Y = 2, X = 3, …., C = 24, B = 25, Z = 26. So, GO = 20 + 12 = 32 and SHE = 8 + 19 + 22 = 49. Similarly, SOME = S + O + M + E = 8 + 12 + 14 + 22 = 56. 8. If AT = 20, BAT = 40, then CAT will be equal to - (a) 30 (b) 50 (c) 60 (d) 70 Sol. (c). Taking A = 1, B = 2, …., T = 20, …., Z = 26, we have : AT = A × T = 1 × 20 = 20 ; BAT = B × A × T = 2 × 1 × 20 = 40. So, CAT = C × A × T = 3 × 1 × 20 = 60. 9. If ZIP = 198 and ZAP = 246, then how will you code VIP ? (a) 174 (b) 222 (c) 888 (d) 990 Sol. (b). Taking Z = 2, Y = 3, …., N = 14, …., B = 26, A = 27, we have : ZIP = (Z + I + P) × 6 = (2 + 19 + 12) × 6 = 33 × 6 = 198. So, VIP = (V + I + P) × 6 = (6 + 19 + 12) × 6 = 37 × 6 = 222. 10. If DEER = 12215 and HIGH = 5645, how will you code HEEL ? (a) 2328 (b) 3449 (c) 4337 (d) 5229 Sol. (d). Clearly, each letter is coded by a numeral which is 3 less than the numeral denoting the position of the letter in the English alphabet. The code for the word is obtained by joining together physically the number codes for the individual letters. H, E, L are 8th, 5th and 12th letters respectively. So, their codes are 5, 2 and 9 respectively. Thus, the code for HEEL becomes 5229.
  • 16. Slide 16 of 42 11. If E = 5 and HOTEL = 12, how will you code LAMB ? (a) 7 (b) 10 (c) 26 (d) 28 Sol. (a). We have : A = 1, B = 2, C = 3, …., Y = 25, Z = 26. Clearly, the code for a word is obtained by dividing the sum of the individual values of its letters by the number of letters in the word. 12. If ZEBRA can be written as 2652181, how can COBRA be written ? (a) 302181 (b) 3152181 (c) 31822151 (d) 1182153 Sol. (b). Putting A = 1, B = 2, C = 3, ….., X = 24, Y = 25, Z = 26, we have : ZEBRA → Z/E/B/R/A → 26/5/2/18/1 → 2652181. COBRA → C/O/B/R/A → 3/15/2/18/1 → 3152181. H + O + T + E + L Thus, HOTEL = 5 8 15 20 5 12 60 12. 5 5 L + A + M + B 12 1 13 2 So, LAMB = 4 4 28 7. 4 + + + + = = = + + + = = =
  • 17. Slide 17 of 42 13. If WORK is coded as 4 − 12 − 9 − 16, then how will you code WOMAN ? (a) 4 − 12 − 14 − 26 −13 (b) 4 − 26 − 14 − 13 − 12 (c) 23 − 12 − 26 − 14 − 13 (d) 23 − 15 − 13 − 1 − 14 (e) None of these Sol. (a). Clearly, each letter is coded by the numeral obtained by subtracting from 27, the numeral denoting the position of the letter in the English alphabet. W, O, M, A, N and 23rd, 15th, 13th, 1st and 14th letters. So, their codes are (27 − 23), (27 − 15), (27 − 13), (27 − 1), (27 − 14) i.e. 4, 12, 14, 26, 13 respectively. 14. If ACNE can be coded as 3, 7, 29, 11, then BOIL will be coded as - (a) 5, 29, 19, 27 (b) 5, 29, 19, 25 (c) 5, 31, 21, 25 (d) 5, 31, 19, 25 Sol. (d). In the given coding system, we have : So, the code for BOIL is 5, 31, 19, 25. Letter A B C D E F G H I J K L M N O Code 3 5 7 9 11 13 15 17 19 21 23 25 27 29 31 15. In O = 16, FOR = 42, then what is FRONT equal to ? (a) 61 (b) 65 (c) 73 (d) 78 Sol. (d). We have : A = 2m B = 3, …., Z = 27. Then, FOR = F + O + R = 7 + 16 + 19 = 42. FRONT = F + R + O +N + T = 7 + 19 + 16 + 15 + 21 = 78.
  • 18. Slide 18 of 42 Type - 5 Directions : The number-group in each question below is to be codified according to the following letter codes : Number 5 1 3 0 2 4 8 7 6 9 Letter code X L M P D B E F K J You have to find out which of the answers (a), (b), (c) or (d) has the correct coded form of the given number group. If none of the coded forms is correct, mark (e) i.e. ‘None of these’ as your answer. 1. 173846 (a) LFMEKB (b) LMFEBK (c) LFMEBK (d) LFEMBK (e) None of these Sol. (c). 1 is coded as L, 7 as F, 3 as M, 8 as E, 4 as B, 6 as K. So, the code for 173846 is LFMEBK. 2. 862941 (a) EDKJBL (b) EKDJLB (c) EKJDBL (d) EKDJBL (e) None of these Sol. (d). 8 is coded as E, 6 as K, 2 as D, 9 as J, 4 as B, 1 as L. So, the code for 862941 is EKDJBL.
  • 19. Slide 19 of 42 3. 430675 (a) BMKPFX (b) BMPKFX (c) BMPKXF (d) BMPFKX (e) None of these Sol. (b). 4 is coded as B, 3 as M, 0 as P, 6 as K, 7 as F, 5 as X. So, the code for 430675 is BMPKFX. 4. 790853 (a) FJPEXM (b) FPJEXM (c) FJPEMX (d) FPJEMX (e) None of these Sol. (a). 7 is coded as F, 9 as J, 0 as P, 8 as E, 5 as X, 3 as M. So, the code for 790853 is FJPEXM. 5. 901273 (a) JPLDFM (b) JPDLFM (c) JLPDFM (d) JPLDMF (e) None of these Sol. (a). 9 is coded as J, 0 as P, 1 as L, 2 as D, 7 as F, 3 as M. So, the code for 901273 is JPLDFM.
  • 20. Slide 20 of 42 Directions : In each of the questions given below, a group of digits followed by four combinations of letter codes labeled (a), (b), (c) and (d) are given. You have to find out which of the combinations is correct coded form of the group of digits as per the following codes and conditions. If none of these four combinations in correct, give (e) i.e. ‘None of these’ as the answer. Digit 9 3 2 8 1 5 7 6 4 0 Latter code M E B N K R H T D J 6. 328496 (a) YBNDTM (b) XBNDTX (c) EBNTDM (d) YBNDTY (e) None of these Sol. (e). 3 is coded as E, 2 as B, 8 as N, 4 as D, 6 as T, 9 as M. So, the code for 328469 is EBNDTM. 7. 631420 (a) TEKBDW (b) TEKDBW (c) TEKDBJ (d) TKDEBW (e) None of these Sol. (b). Since the last digit is 0, so 0 shall be coded as W. 6 is coded as T, 3 as E, 1 as K, 4 as D, 2 as B. So, the code for 631420 is TEKDBW. Conditions : (i) If the first digit is an odd number and the last digit is an even number, both are to be coded as ‘X’. (ii) If the first digit is an even number and the last digit is an odd number, both are to be coded as ‘Y’. (iii) If either the first or the last digit is 0, then 0 is to be coded as ‘W’.
  • 21. Slide 21 of 42 8. 640598 (a) TDJRMN (b) TDWRMN (c) XDJRMN (d) TDJRMY (e) None of these Sol. (a). 6 is coded as T, 4 as D, 0 as J, 5 as R, 9 as M, 8 as N. So, the code for 640598 is TDJRMN. 9. 893561 (a) XMERTY (b) NMERTK (c) XMERTX (d) YMERTY (e) None of these Sol. (d). Since the given number-group begins with an even number and ends with an odd number, so both the first and last digits i.e. 8 and 1 shall be coded as Y. 9 is coded as M, 3 as E, 5 as R, 6 as T. So, the code for 893561 is YMERTY.10. 743506 (a) XDERWT (b) HDERWT (c) XDERJX (d) YDERJY (e) None of these Sol. (c). Since the first digit is an odd number and the last digit is an even number, so both 7 and 6 shall be coded as X. 4 is coded as D, 3 as E, 5 as R, 0 as J. So, the code for 743506 is XDERJX.
  • 22. Slide 22 of 42 Directions : Study the following information carefully to answer these questions : The digits from 0 to 9 are coded as shown below along with the exceptions I and II : Digit 3 8 0 7 4 6 9 2 5 Code H $ R A M % L K ξ 11. What will be the code for 764981 ? (a) A%ML$ξ (b) Y%ML$ξ (c) Y%ML$# (d) A%ML$# (e) None of these Sol. (c). Since the number begins and ends with an odd digit, so 7 shall be coded as Y and 1 as #. The codes for 6, 4, 9, 8 are %, M, L, $ respectively. So, the required code is Y%ML$#. 12. What dose MHLEK% represent ? (a) 439526 (b) 439520(c) 436529 (d) 489526 (e) None of these Sol. (a). M, H, L, E, K, % are codes for 4, 3, 9, 5, 2, 6 respectively. So, MHLEK% represents 439526. Exception I : If a number begins and ends with a non-zero odd digit, then the first and the last digits are to be coded as Y and # respectively. Exception II : If a number begins and ends with an even digit (including zero), then the first and the last digits are to be coded as β and X respectively.
  • 23. Slide 23 of 42 13. What will be the code for 278140 ? (a) βA$ξMR (b) βA$ξMX (c) KA$ξMR (d) YA$ξM# (e) None of these Sol. (b). Since 27 8140 has even digits at the first and last places, so 2 shall be coded as β and 0 as X. The codes for 7, 8, 1, 4 are A, $, ξ, M respectively. So, the required code is βA$ξMX 14. What will be the code for 173548 ? (a) ξAHEK$ (b) ξAREM$ (c) ξAHEM$ (d) ξAHME$ (e) None of these Sol. (c). 1 is coded as ξ, 7 as A, 3 as H, 5 as E, 4 as M, 8 as $. So, the code for 1735 48 is ξAHEM$. 15. What does R%LAKξ represent ? (a) 069725 (b) 697210 (c) 069751 (d) 064721 (e) None of these Sol. (e). R, %, L, A, K and ξ are codes for 0, 6, 9, 7, 2 and 1 respectively. So, R%LAKξ represents 069721.
  • 24. Slide 24 of 42 Type - 6 Directions : In each of the following questions, a word is represented by only one set of numbers as given in any one of the alternatives. The sets of numbers given in the alternatives are represented by two classes of alphabets as in the two given matrices. The columns and rows of Matrix I are numbers from 0 to 4 and those of Matrix II from 5 to 9. A letter from these matrices can be represented first by its row and then the column number e.g., in the matrices for questions 1 to 4, M can be represented by 14, 21, etc ; O can be represented by 20, 32 etc. Similarly you have to identify the correct set for the word given in each question. Matrix I 0 1 2 3 4 0 F O M S R 1 S R F O M 2 O M S R F 3 R F O M S 4 M S R F O Matrix II 5 6 7 8 9 5 A T D I P 6 I P A T D 7 T D I P A 8 P A T D I 9 D I P A T (Questions)
  • 25. Slide 25 of 42 1. MOST (a) 40, 44, 22, 89 (b) 33, 20, 11, 79 (c) 21, 00, 03, 88 (d) 02, 13, 34, 56 Sol. (d). From matrix I, M can be coded as 02, 14, 21, 33 or 40. From matrix I, O can be coded as 01, 13, 20, 32 or 44. From matrix I, S can be coded as 03, 10, 22, 34 or 41. From matrix II, T can be coded as 56, 68, 75, 87 or 99. Clearly, (d) is the only set of correct codes. 2. ROAD (a) 42, 32, 79, 58 (b) 23, 32, 98, 99 (c) 11, 13, 67, 69 (d) 04, 20, 55, 78 Sol. (c). From matrix I, R can be coded as 04, 11, 23, 30 or 42. From matrix I, O can be coded as 01, 13, 20, 32 or 44. From matrix II, A can be coded as 55, 67, 79, 86 or 98. From matrix II, D can be coded as 57, 69, 76, 88 or 95. Clearly, (c) contains the correct codes. 3. STOP (a) 10, 56, 44 , 97 (b) 41, 68, 01, 77 (c) 22, 75, 32, 86 (d) 33, 99, 42, 59 Sol. (a). From matrix I, S can be coded as 03, 10, 22, 34 or 41. From matrix II, T can be coded as 56, 68, 75, 87 or 99. From matrix I, O can be coded as 01, 13, 20, 32 or 44. From matrix II, P can be coded as 59, 66, 78, 85 or 97. 4. FOAM (a) 24, 01, 55, 22 (b) 00, 01, 67, 33 (c) 12, 13, 67, 23 (d) 43, 52, 56, 33 Sol. (b). From matrix I, F can be coded as 00, 12, 24, 31 or 43. From matrix I, O can be coded as 01, 13, 20, 32 or 44. From matrix II, A can be coded as 55, 67, 79, 86 or 98. From matrix I, M can be coded as 02, 14, 21, 33 or 40.
  • 26. Slide 26 of 42 Matrix I 0 1 2 3 4 0 A E S T H 1 T H A E S 2 E S T H A 3 H A E S T 4 S T H A E Matrix II 5 6 7 8 9 5 P O R K L 6 K L P O R 7 O R K L P 8 L P O R K 9 R K L P O (Questions) 5. EAST (a) 44, 32, 21, 03 (b) 32, 31, 02, 04 (c) 20, 43, 33, 11 (d) 13, 12, 14, 10 Sol. (d). From matrix I, E can be coded as 01, 13, 20, 32 or 44. From matrix I, A can be coded as 00, 12, 24, 31 or 43. From matrix I, S can be coded as 02, 14, 21, 33 or 40. From matrix I, T can be coded as 03, 10, 22, 34 or 41. 6. ROSE (a) 95, 75, 02, 32 (b) 88, 76, 31, 32 (c) 86, 67, 33, 44 (d) 57, 87, 32, 33 Sol. (a). From matrix II, R can be coded as 57, 69, 76, 88 or 95. From matrix II, O can be coded as 56, 68, 75, 87 or 99. From matrix I, S can be coded as 02, 14, 21, 33 or 40. From matrix I, E can be coded as 01, 13, 20, 32 or 44.
  • 27. Slide 27 of 42 7. SOLE (a) 41, 57, 87, 31 (b) 33, 99, 66, 44 (c) 21, 75, 44, 02 (d) 02, 78, 87, 13 Sol. (b). From matrix I, S can be coded as 02, 14, 21, 33 or 40. From matrix II, O can be coded as 56, 68, 75, 87 or 99. From matrix II, L can be coded as 59, 66, 78, 85 or 97. From matrix I, E can be coded as 01, 13, 20, 32 or 44. 8. LAKE (a) 97, 00, 77, 12 (b) 66, 12, 58, 40 (c) 85, 31, 77, 44 (d) 77, 43, 76, 31 Sol. (c). From matrix II, L can be coded as 59, 66, 78, 85 or 97. From matrix I, A can be coded as 00, 12 24, 31 or 43. From matrix II, K can be coded as 58, 65, 77, 89 or 96. From matrix I, E can be coded as 01, 13, 20, 32 or 44. 9. LEST (a) 97, 32, 21, 34 (b) 87, 32, 21, 31 (c) 85, 02, 04, 22 (d) 66, 00, 20, 34 Sol. (a). From matrix II, L can be coded as 59, 66, 78, 85 or 97. From matrix I, E can be coded as 01, 13, 20, 32 or 44. From matrix I, S can be coded as 02, 14, 21, 33 or 40. From matrix I, T can be coded as 03, 10, 22, 34 or 41.
  • 28. Slide 28 of 42 Matrix I 0 1 2 3 4 0 F A N O I 1 I O F A N 2 A N O I F 3 O F I N A 4 N I A F O Matrix II 5 6 7 8 9 5 S E H B T 6 H S E T B 7 B T S E H 8 E H T B S 9 T S E H B (Questions) 10. NEST (a) 02, 56, 55, 59 (b) 14, 67, 66, 67 (c) 21, 76, 77, 76 (d) 33, 85, 88, 86 Sol. (a). From matrix I, N can be coded as 02, 14, 21, 33 or 40. From matrix II, E can be coded as 56, 67, 78, 85 or 97. From matrix II, S can be coded as 55, 66, 77, 89 or 96. From matrix II, T can be coded as 59, 68, 76, 87 or 95. 11. FAITH (a) 43, 42, 41, 78, 89 (b) 31, 34, 23, 76, 79 (c) 24, 31, 10, 59, 57 (d) 12, 20, 40, 68, 65 Sol. (b). From matrix I, F can be coded as 00, 12, 24, 31 or 43. From matrix I, A can be coded as 01, 13, 20, 34 or 42. From matrix I, I can be coded as 04, 10, 23, 32 or 41. From matrix II, T can be coded as 59, 68, 76, 87 or 95. From matrix II, H can be coded as 57, 65, 79, 86 or 98.
  • 29. Slide 29 of 42 12. FINE (a) 31, 32, 33, 82 (b) 24, 19, 21, 78 (c) 12, 10, 13, 67 (d) 00, 04, 02, 56 Sol. (d). From matrix I, F can be coded as 00,12, 24, 31 or 43. From matrix I, I can be coded as 04, 10, 23, 32 or 41. From matrix I, N can be coded as 02, 14, 21, 33 or 40. From matrix II, E can be coded as 56, 67, 78, 85 or 97. 13. HEAT (a) 79, 53, 20, 87 (b) 65, 56, 13, 57 (c) 57, 56, 01, 59 (d) 29, 85, 34, 93 Sol. (c). From matrix II, H can be coded as 57, 65, 79, 86 or 98. From matrix II, E can be coded as 56, 67, 78, 85 or 97. From matrix I, A can be coded as 01, 13, 20, 34 or 42. From matrix II, T can be coded as 59, 68, 76, 87 or 95. 14. BOTH (a) 88, 30, 85, 86 (b) 75, 22, 76, 79 (c) 69, 67, 68, 59 (d) 58, 02, 68, 65 Sol. (b). From matrix II, B can be coded as 58, 69, 75, 88 or 99. From matrix I, O can be coded as 03, 11, 22, 30 or 44. From matrix II, T can be coded as 59, 68, 76, 87 or 95. From matrix II, H can be coded as 57, 65, 79, 86 or 98.
  • 30. Slide 30 of 42 Matrix I 0 1 2 3 4 0 D O B A I 1 O B A I D 2 B A I D O 3 A I D O B 4 I D O B A Matrix II 5 6 7 8 9 5 W N R M L 6 N R M L W 7 R M L W N 8 M L W N R 9 L W N R M (Questions) 15. DRAW (a) 41, 66, 23, 55 (b) 32, 75, 44, 76 (c) 23, 57, 30, 68 (d) 14, 89, 12, 78 Sol. (d). From matrix I, D can be coded as 00, 14, 23, 32 or 41. From matrix II, R can be coded as 57, 66, 75, 89 or 98. From matrix I, A can be coded as 03, 12, 21, 30 or 44. From matrix II, W can be coded as 55, 69, 78, 87 or 96.
  • 31. Slide 31 of 42 Type - 7 1. If ‘white’ is called ‘blue’, ‘blue’ is called ‘red’, ‘red’ is called ‘yellow’, ‘yellow’ is called ‘green’, ‘green’ is called ‘black’, ‘black’ is called ‘violet’ and ‘violet’ is called ‘orange’, what would be the colour of human blood ? Sol. (c). The colour of the human blood is ‘red’ and as given, ‘red’ is called ‘yellow’. So, the colour of human blood is ‘yellow’. 3. If ‘pen’ is ‘table’, ‘table’ is ‘fan’, ‘fan’ is ‘chair’ and ‘chair’ is ‘roof’, on which of the following will a person sit ? Sol. (c). A person will sit on a ‘chair’ but a ‘chair’ is called ‘roof’. So, a person will sit on the ‘roof’. 4. If ‘bat’ is ‘racket’, ‘racket’ is ‘football’, ‘football’ is ‘shuttle’, ‘shuttle’ is ‘ludo’ and ‘ludo’ is ‘carrom’, what is cricket played with ? Sol. (a). Cricket is played with a ‘bat’ and ‘bat’ is called ‘racket’. So, cricket is played with a ‘racket’. (a) Red (b) Green (c) Yellow (d) Violet (e) Orange 2. If ‘oranges’ are ‘apples’, ‘bananas’ are ‘apricots’, ‘apples’ are ‘chillies’, ‘apricots’ are ‘oranges’, and ‘chillies’ are ‘bananas’, then which of the following are green in colour ? Sol. (d). ‘Chillies’ are green in colour and as given, ‘chillies’ are ‘bananas’. So, ‘bananas’ are green in colour. (a) Apricots (b) Apples (c) Chillies (d) Bananas (e) Oranges (a) Fan (b) Chair (c) Roof (d) Table (e) Pen (a) Racket (b) Football (c) Bat (d) Shuttle (e) Carrom
  • 32. Slide 32 of 42 5. If ‘sky’ is ‘star’, ‘star’ is ‘cloud’, ‘cloud’ is ‘earth’, ‘earth’ is ‘tree’ and ‘tree’ is ‘book’, then where do the birds fly ? Sol. (c). Birds fly in the ‘sky’ and as given, ‘sky’ is ‘star’. So, birds fly in the ‘star’. 7. If ‘book’ is called ‘watch’, ‘watch’ is called ‘bag’, ‘bag’ is called ‘dictionary’ and ‘dictionary’ is called ‘window’, which is used to carry the books ? Sol. (a). Clearly, a ‘bag’ is used to carry the books but a ‘bag’ is called ‘dictionary’. So, a ‘dictionary’ will be used to carry the books. 8. If ‘cushion’ is called ‘pillow’, ‘pillow’ is called ‘mat’, ‘mat’ is called ‘bed sheet’ and ‘bed sheet’ is called ‘cover’, which will be spread on the floor ? Sol. (b). ‘Mat’ will be spread on the floor. But ‘mat’ is called ‘bed sheet’. So, a man will sleep on the ‘window’. (a) Cloud (b) Sky (c) Star (d) Data inadequate (e) None of these 6. If ‘room’ is called ‘bed’, ‘bed’ is called ‘window’, ‘window’ is called ‘flower’ and ‘flower’ is called ‘cooler’, on what would a man sleep ? Sol. (a). A man sleeps on a ‘bed’ and as given, ‘bed’ is called ‘window’. So, a man will sleep on the ‘window’. (a) Window (b) Bed (c) Flower (d) Cooler (e) None of these (a) Dictionary (b) Bag (c) Book (d) Watch (e) None of these (a) Cover (b) Bed sheet (c) Mat (d) Pillow (e) None of these
  • 33. Slide 33 of 42 9. If ‘orange’ is called ‘butter’, ‘butter’ is called ‘soap’, ‘soap’ is called ‘ink’, ‘ink’ is called ‘honey’ and ‘honey’ is called ‘orange’, which of the following is used for washing clothes ? Sol. (e). Clearly, ‘soap’ is used for washing the clothes. But, ‘soap’ is called ‘ink’. So, ‘ink’ is used for washing the clothes. 11. If ‘bangle’ is called ‘cassette’, ‘cassette’ is called ‘table’, ‘table’ is called ‘game’ and ‘game’ is called ‘cupboard’, then which is played in the tape recorder ? Sol. (c). Clearly, a ‘cassette’ is played in the tape-recorder. But a ‘cassette’ is called ‘table’. So, a ‘table’ will be played in the tape-recorder. 12. If ‘black’ means ‘pink’, ‘pink’ means ‘blue’, ‘blue’ means ‘white’, ‘white’ means ‘yellow’, ‘yellow’ means ‘red’ and ‘red’ means ‘brown’, then what is the colour of clear sky ? Sol. (d). The colour of clear sky is ‘blue’. But, as given, ‘pink’ means ‘blue’. So, the colour of clear sky is ‘pink’. (a) Honey (b) Butter (c) Orange (d) Soap (e) Ink 10. If ‘sand’ is called ‘air’, ‘air’ is called ‘plateau’, ‘plateau’ is called ‘well’, ‘well’ is called ‘island’ and ‘island’ is called ‘sky’, then from where will a woman draw water ? Sol. (b). A woman shall draw water from a ‘well’ but a ‘well’ is called ‘island’. So, the woman will draw water from an ‘island’. (a) Well (b) Island (c) Sky (d) Air (e) None of these (a) Bangle (b) Cassette (c) Table (d) Cupboard (e) None of these (a) Brown (b) Red (c) Blue (d) Pink (e) None of these
  • 34. Slide 34 of 42 13. If ‘rain’ is ‘water’, ‘water’ is ‘road’, ‘road’ is ‘cloud’, ‘cloud’ is ‘sky’, ‘sky’ is ‘sea’ and ‘sea’ is ‘path’, where do aeroplanes fly Sol. (b). The aeroplanes fly in the ‘sky’ and the ‘sky’ is called ‘sea’. So, the aeroplanes fly in the ‘sea’. 15. If ‘dust’ is called ‘air’, ‘air’ is called ‘fire’, ‘fire’ is called ‘water’, ‘water’ is called ‘colour’, ‘colour’ is called ‘rain’ and ‘rain’ is called ‘dust’, then where do fish live ? Sol. (c). Fishes live in ‘water’ and as given, ‘water’ is called ‘colour’. So, fishes live in ‘colour’. (a) Road (b) Sea (c) Cloud (d) Water (e) None of these 14. If ‘water’ is called ‘food’, ‘food’ is called ‘tree’, ‘tree’ is called ‘sky’, ‘sky’ is called ‘wall’ on which of the following grows a fruit ? Sol. (c). A fruit grows on a ‘tree’ and ‘tree’ is called ‘sky’. So, a fruit grows on the ‘sky’. (a) Water (b) Food (c) Sky (d) Tree (e) Wall (a) Fire (b) Water (c) Colour (d) Dust
  • 35. Slide 35 of 42 Type - 8 1. In a certain code language, ‘col tip mot’ means ‘singing is appreciable’, ‘mot baj min’ means ‘dancing is good’ and ‘tip nop baj’ means ‘singing and dancing’, which of the following means ‘good’ in that code language ? Sol. (b). In the first and second statements, the common code-word is ‘mod’ and the common word is ‘is’. So, ‘mot’ means ‘is’. In the second and third statements, the common code-word is ‘baj’ and the common word is ‘dancing’. So, ‘baj’ means ‘dancing’. Thus, in the second statement, ‘min’ means ‘good’. (a) not (b) min (c) baj (d) Cannot be determined (e) None of these 2. In a certain code language, ‘mink yang pe’ means ‘fruits are ripe’, ‘pe lao may mink’ means ‘oranges are not ripe’ and ‘may pe nue mink’ means ‘mangoes are not ripe’. Which word in that language means ‘mangoes’ ? Sol. (c). In the second and third statements, the common code-words are ‘pe’, ‘mink’ and ‘may’ and the common words are ‘are’, ‘not’ and ‘ripe’. So, in the third statement, ‘nue’ means ‘mangoes’. (a) may (b) pe (c) nue (d) mink
  • 36. Slide 36 of 42 3. In a certain code language, ‘tom kun sud’ means ‘dogs are barking’, ‘kun jo mop’ means ‘dogs and horses’ and ‘mut tom ko’ means ‘donkeys are mad’. Which word in that language means ‘barking’ ? Sol. (a). In the first and second statements, the common code-word is ‘kun’ and the common word is ‘dogs’. So, ‘kun’ means ‘dogs’. In the first and third statements, the common code- word is ‘tom’ and the common word is ‘are’. So, ‘tom’ means ‘are’. Thus, in the first statement, ‘sud’ means ‘barking’. (a) sud (b) kun (c) jo (d) tom (e) ko 4. In a code language, ‘mok dan sil’ means ‘nice big house’, ‘fit kon dan’ means ‘house is good’ and ‘warm tir fit’ means ‘cost is hogh’. Which word stands for ‘good’ in that language ? Sol. (d). In the first and second statements, the common code-word is ‘dan’ and the common word is ‘house’. So, ‘dan’ means ‘house’. In the second and third statements, the common code-word is ‘fit’ and the common word is ‘is’. So, ‘fit’ means ‘is’. Thus, in the second statement, ‘kon’ stands for ‘good’. (a) mok (b) dan (c) fit (d) kon
  • 37. Slide 37 of 42 5. If ‘ski rps tri’ stands for ‘nice Sunday morning’, ‘the sti rps’ stands for ‘every Tuesday morning’ and ‘ski ptr qlm’ stands for ‘nice market place’, which word stands for ‘Sunday’ ? Sol. (c). In the first and second statements, the common code-word is ‘rps’ and the common word is ‘morning’. So, ‘rps’ stands for ‘morning’. In the first and third statements, the common code- word is ‘ski’ and the common word is ‘nice’. So, ‘ski’ stands for ‘nice’. Thus in the second statement, ‘tri’ stands for ‘Sunday’. (a) vog (b) nat (c) dor (d) bis (e) None of these 6. In a certain language, ‘pre nat bis’ means ‘smoking is harmful’, ‘vog dor nat’ means ‘avoid harmful habit’ and ‘dor bis yel’ means ‘please avoid smoking’. Which of the following means ‘habit’ in that language ? Sol. (a). In the first and second statements, the common code-word is ‘nat’ and the common word is ‘harmful’. So, ‘nat’ means ‘harmful’. In the second and third statements, the common code-word is ‘dor ’ and the common word is ‘avoid’. So, ‘dor’ means ‘avoid’. Thus, in the second statement, ‘vog’ means ‘habit’. (a) ski (b) rps (c) tri (d) qlm
  • 38. Slide 38 of 42 7. In a certain coding system, ‘rbm std bro pus’ means ‘the cat is beautiful’, ‘tnh pus dim std’ means ‘the dog is brown’, ‘pus dim bro pus cus’ means ‘the dog has the cat’. What is the code for ‘has’ ? Sol. (d). In the third statements, the common code-word ‘pus’ occurs twice and the word ‘the’ also occurs twice. So, the code-word for ‘the’ is ‘pus’. Now, in the first and third statements, the common code-word ‘pus’ stands for ‘the’. So, the other common code-word ‘bro’ stands for the other common word i.e. ‘cat’. Similarly, in the second and third statements, the common code-word ‘dim’ stands for the common word ‘dog’. Thus, in the third statement, the remaining code- word i.e. ‘cus’ stands for ‘has’. (a) lon (b) sig (c) fin (d) None of these 8. In a certain code language, ‘put tir fin’ means ‘delicious juicy fruit’, ‘tie dip sig’ means ‘beautiful while lily’ and ‘sig lon fin’ means ‘lily and fruit’. Which of the following stands for ‘and’ in that language ? Sol. (a). In the first and third statements, the common code- word i.e. ‘fin’ and the common word is ‘fruit’. So, ‘fin’ stands for ‘fruit’. In the second and third statements, the common code-word is ‘sig’ and the common word is ‘lily’. So, ‘sig’ stands for ‘lily’. Thus, in the third statement, ‘lon’ stands for ‘and’. (a) std (b) dim (c) bro (d) cus
  • 39. Slide 39 of 42 9. In a certain code language, ‘dom pul ta’ means ‘bring hot food’, ‘pul tir sop’ means ‘food is good’ and ‘tak da sop’ means ‘good bright boy’. Which of the following does mean ‘hot’ in that language ? Sol. (d). We can find the code for ‘food’ from the first and second statements. Now, to find the code for ‘hot’, we need the code for ‘bring’ which cannot be determined from the given information. (a) ja (b) ma (c) kop (d) top (e) ki 10. In a certain code language, ‘po ki top ma’ means ‘Usha is playing cards’, ‘kop ja ki ma’ means ‘Asha is playing tennis’, ‘ki top sop ho’ means ‘they are playing football’ and ‘po sur kop’ means ‘cards and tennis’. Which word in that language means ‘Asha’ ? Sol. (a). In the first and second statements, the common code-words are ‘ki’ and ‘ma’ and the common words are ‘is’ and ‘playing’. So, ‘ki’ and ‘ma’ are the codes for ‘is’ and ‘playing’. In the second and fourth statements, the common code-word is ‘kop’ and the common word is ‘tennis’. So, ‘kop’ stands for ‘tennis’. Thus, in the second statement, ‘ja’ stands for ‘Asha’. (a) dom (b) pul (c) ta (d) Cannot be determined (e) None of these
  • 40. Slide 40 of 42 11. If ‘cinto baoli tsi nzro’ means ‘her village is Sarurpur’, ‘mhi cinto keepi tsi oind’ means ‘her first love is literature’ and ‘oind geit tsi cinto pki’ means ‘literature collection is her hobby’, which word would mean ‘literature’ ? Sol. (c). In the first and second statements, the common code-words are ‘cinto’ and ‘tsi’ and the common words are ‘her’ and ‘is’. So, ‘cinto’ and ‘tsi’ are the codes for ‘her’ and ‘is’. In the second and third statements, the common code-words are ‘cinto’, ‘tsi’ and ‘oind’ and the common words are ‘her’, ‘is’ and ‘literature’. Now, ‘cinto’ and ‘tsi’ are codes for ‘her’ and ‘is’. So, ‘oind’ stands for ‘literature’. (a) cinto (b) baoli (c) oind (d) geit
  • 41. Slide 41 of 42 Directions : (Questions) In a certain code language, (A) ‘pit dar na’ means ‘you are good’. (B) ‘dar tok pa’ means ‘good and bad’. (C) ‘tim na tok’ means ‘they are bad’. 12. In that language, which word stands for ‘they’ ? Sol. (c). In statements (A) and (C), the common code-word is ‘na’ and the common word is ‘are’. So, ‘na’ stands for ‘are’. In statements (B) and (C), the common code-word is ‘tok’ and the common word is ‘bad’. So, ‘tok’ stands for ‘bad’. Thus, in the statement (C), ‘tim’ stands for ‘they’. (a) na (b) tok (c) tim (d) pit (e) None of these 13. To find the answer to the above question, which of the given statements can be dispensed with ? Sol. (e). Since all the given statements have been used to find the answer, so none of the given statements can be dispensed with. (a) Only A (b) Only B (c) A or B (d) B and C (e) None of these
  • 42. Slide 42 of 42 Directions : (Questions) In a certain code language, (A) ‘pic vic nic’ means ‘winter is cold. (B) ‘to nic re’ means ‘summer is hot’. (C) ‘re pic boo’ means ‘winter and summer’. (D) ‘vic tho pa’ means ‘nights are cold’. 14. Which word in that language means ‘summer’ ? Sol. (b). In statements (B) and (C), the common code-word is ‘re’ and the common word is ‘summer’. So,‘re’ means ‘summer’. (a) nic (b) re (c) to (d) pic (e) vic 15. Which of the given statements is superfluous and can be dispensed with while answering the above question ? Sol. (c). Clearly, both statements (A) and (D) are superfluous. (a) Only A (b) Only D (c) Both A and D (d) Neither A nor D (e) None of these